You are on page 1of 65

Philippine Health Care Providers v CIR G.R. No. 167330 June 12, 2008 J.

Corona Facts: The petitioner, a prepaid health-care organization offering benefits to its members. The CIR found that the organization had a deficiency in the payment of the DST under Section 185 of the 1997 Tax Code which stipulated its implementation: On all policies of insurance or bonds or obligations of the nature of indemnity for loss, damage, or liability made or renewed by any person, association or company or corporation transacting the business of accident, fidelity, employer's liability, plate, glass, steam boiler, burglar, elevator, automatic sprinkler, or other branch of insurance (except life, marine, inland, and fire insurance) The CIR sent a demand for the payment of deficiency taxes, including surcharges and interest, for 1996-1997 in the total amount of P224,702,641.18. The petitioner protested to the CIR, but it didnt act on the ap peal. Hence, the company had to go to the CTA. The latter declared judgment against them and reduced the taxes. It ordered them to pay 22 million pesos for deficiency VAT for 1997 and 31 million deficiency VAT for 1996. CA denied the companys appeal an d increased taxes to 55 and 68 million for 1996 to 1997. Issues: WON a health care agreement in the nature of an insurance contract and therefore subject to the documentary stamp tax (DST) imposed under Section 185 of Republic Act 8424 (Tax Code of 1997) Held: Yes. Petition dismissed. Ratio: The DST is levied on the exercise by persons of certain privileges conferred by law for the creation, revision, or termination of specific legal relationships through the execution of specific instruments. The DST is an excise upon the privilege, opportunity, or facility offered at exchanges for the transaction of the business. In particular, the DST under Section 185 of the 1997 Tax Code is imposed on the privilege of making or renewing any policy of insurance (except life, marine, inland and fire insurance), bond or obligation in the nature of indemnity for loss, damage, or liability. Petitioner's health care agreement is primarily a contract of indemnity. And in the recent case of Blue Cross Healthcare, Inc. v. Olivares, this Court ruled that a health care agreement is in the nature of a non-life insurance policy. Its health care agreement is not a contract for the provision of medical services. Petitioner does not actually provide medical or hospital services but merely arranges for the same It is also incorrect to say that the health care agreement is not based on loss or damage because, under the said agreement, petitioner assumes the liability and indemnifies its member for hospital, medical and related expenses (such as professional fees of physicians). The term "loss or damage" is broad enough to cover the monetary expense or liability a member will incur in case of illness or injury. Philamcare Health Systems, Inc. v. CA.- The health care agreement was in the nature of non-life insurance, which is primarily a contract of indemnity. Similarly, the insurable interest of every member of petitioner's health care program in obtaining the health care agreement is his own health. Under the agreement, petitioner is bound to indemnify any member who incurs hospital, medical or any other expense arising from sickness, injury or other stipulated contingency to the extent agreed upon under the contract. Phil. American Life Insurance Company v. Ansaldo (1994) FACTS: Ramon M. Paterno, Jr. sent a letter dated April 17, 1986 to Insurance Commissioner alleging certain problems encountered by agents, supervisors, managers and public consumers of the Philippine American Life Insurance Company (Philamlife) During the hearing Ramon stated that the contract of agency is illegal Philamlife through its president De los Reyes contended that the Insurance Commissioner as a quasijudicial body cannot rule on the matter ISSUE: 1. W/N the Insurance Commissioner has the authority to regulate the business of insurance - YES

2. W/N the business of insurance covers the contract of agency - NO HELD: petition is GRANTED 1. YES.

Insurance Code Sec. 414 Sec. 414. The Insurance Commissioner shall have the duty to see that all laws relating to insurance, insurance companies and other insurance matters, mutual benefit associations, and trusts for charitable uses are faithfully executed and to perform the duties imposed upon him by this Code, and shall, notwithstanding any existing laws to the contrary, have sole and exclusive authority to regulate the issuance and sale of variable contracts as defined in section two hundred thirty-two and to provide for the licensing of persons selling such contracts, and to issue such reasonable rules and regulations governing the same. The Commissioner may issue such rulings, instructions, circulars, orders and decision as he may deem necessary to secure the enforcement of the provisions of this Code, subject to the approval of the Secretary of Finance. Except as otherwise specified, decisions made by the Commissioner shall be appealable to the Secretary of Finance. Sec. 415 Sec. 415. In addition to the administrative sanctions provided elsewhere in this Code, the Insurance Commissioner is hereby authorized, at his discretion, to impose upon the insurance companies, their directors and/or officers and/or agents, for any willful failure or refusal to comply with, or violation of any provision of this Code, or any order, instruction, regulation, or ruling of the Insurance Commissioner, or any commission or irregularities, and/or conducting business in an unsafe or unsound manner as may be determined by the Insurance Commissioner, the following: (a) fines not in excess of five hundred pesos a day; and (b) suspension, or after due hearing, removal of directors and/or officers and/or agents. Insurance Commissioner has the authority to regulate the business of insurance 2. NO. power does not cover the relationship affecting the insurance company and its agents but is limited to adjudicating claims and complaints filed by the insured against the insurance company While the subject of Insurance Agents and Brokers is discussed under Chapter IV, Title I of the Insurance Code, the provisions of said Chapter speak only of the licensing requirements and limitations imposed on insurance agents and brokers. Great Pacific Life Assurance Corporation v. Judico, 180 SCRA 445 (1989): o insurance company may have two classes of agents who sell its insurance policies: (1) salaried employees who keep definite hours and work under the control and supervision of the company - governed by the Contract of Employment and the provisions of the Labor Code (2) registered representatives, who work on commission basis. - governed by the Contract of Agency and the provisions of the Civil Code on the Agency White Gold v Pioneer G.R. No. 154514. July 28, 2005 Facts: White Gold procured a protection and indemnity coverage for its vessels from The Steamship Mutual through Pioneer Insurance and Surety Corporation. White Gold was issued a Certificate of Entry and Acceptance. Pioneer also issued receipts. When White Gold failed to fully pay its accounts, Steamship Mutual refused to renew the coverage. Steamship Mutual thereafter filed a case against White Gold for collection of sum of money to recover the unpaid balance. White Gold on the other hand, filed a complaint before the Insurance Commission claiming that Steamship Mutual and Pioneer violated provisions of the Insurance Code.

The Insurance Commission dismissed the complaint. It said that there was no need for Steamship Mutual to secure a license because it was not engaged in the insurance business and that it was a P & I club. Pioneer was not required to obtain another license as insurance agent because Steamship Mutual was not engaged in the insurance business. The Court of Appeals affirmed the decision of the Insurance Commissioner. In its decision, the appellate court distinguished between P & I Clubs vis--vis conventional insurance. The appellate court also held that Pioneer merely acted as a collection agent of Steamship Mutual. Hence this petition by White Gold. Issues: 1. Is Steamship Mutual, a P & I Club, engaged in the insurance business in the Philippines? 2. Does Pioneer need a license as an insurance agent/broker for Steamship Mutual? Held: Yes. Petition granted. Ratio: White Gold insists that Steamship Mutual as a P & I Club is engaged in the insurance business. To buttress its assertion, it cites the definition as an association composed of shipowners in general who band together for the specific purpose of providing insurance cover on a mutual basis against liabilities incidental to shipowning that the members incur in favor of third parties. They argued that Steamship Mutuals primary purpose is to solicit and provide protection and indemnity coverage and for this purpose, it has engaged the services of Pioneer to act as its agent. Respondents contended that although Steamship Mutual is a P & I Club, it is not engaged in the insurance business in the Philippines. It is merely an association of vessel owners who have come together to provide mutual protection against liabilities incidental to shipowning. Is Steamship Mutual engaged in the insurance business? A P & I Club is a form of insurance against third party liability, where the third party is anyone other than the P & I Club and the members. By definition then, Steamship Mutual as a P & I Club is a mutual insurance association engaged in the marine insurance business. The records reveal Steamship Mutual is doing business in the country albeit without the requisite certificate of authority mandated by Section 187 of the Insurance Code. It maintains a resident agent in the Philippines to solicit insurance and to collect payments in its behalf. Steamship Mutual even renewed its P & I Club cover until it was cancelled due to non-payment of the calls. Thus, to continue doing business here, Steamship Mutual or through its agent Pioneer, must secure a license from the Insurance Commission. Since a contract of insurance involves public interest, regulation by the State is necessary. Thus, no insurer or insurance company is allowed to engage in the insurance business without a license or a certificate of authority from the Insurance Commission. 2. Pioneer is the resident agent of Steamship Mutual as evidenced by the certificate of registration issued by the Insurance Commission. It has been licensed to do or transact insurance business by virtue of the certificate of authority issued by the same agency. However, a Certification from the Commission states that Pioneer does not have a separate license to be an agent/broker of Steamship Mutual. Although Pioneer is already licensed as an insurance company, it needs a separate license to act as insurance agent for Steamship Mutual. Section 299 of the Insurance Code clearly states: SEC. 299 No person shall act as an insurance agent or as an insurance broker in the solicitation or procurement of applications for insurance, or receive for services in obtaining insurance, any commission or other compensation from any insurance company doing business in the Philippines or any agent thereof, without first procuring a license so to act from the Commissioner.

Filipinas Cia de Seguros vs Christern 89 Phil 54 Fact: On October 1, 1941, the respondent corporation, Christern Huenefeld and Co., Inc., after payment of corresponding premium, obtained from the petitioner, Filipinas Cia de Seguros fire policy covering merchandise contained in a building located at Binondo, Manila. On February 27, 1942 or during the Japanese military occupation, the building and insured merchandise were burned. In due time the respondent submitted to the petitioner its claim under the policy. The petitioner refused to pay the claim on the ground that the policy in favor of the respondent that ceased to be a force on the date the United States declared war against Germany, the respondent corporation (through

organized under and by virtue of the laws of Philippines) being controlled by German subjects and the petitioner being a company under American jurisdiction when said policy was issued on October 1, 1941. The theory of the petitioner is that the insured merchandise was burned after the policy issued in 1941 had ceased to be effective because the outbreak of the war between United States and Germany on December 10, 1941, and that the payment made by the petitioner to the respondent corporation during the Japanese military occupation was under pressure. Issue: W/N a public enemy can be insured. Ruling: Since the majority of stockholders of the respondent corporation were German subjects, the respondent became an enemy of the state upon the outbreak of the war between US and Germany. The English and American cases relied upon by the Court of Appeals lost in force upon the latest decision of the Supreme Court of US in which the control test has adopted. Since World War I, the determination of enemy nationality of corporations has been discussed in many countries, belligerent and neutral. A corporation was subject to enemy legislation when it was controlled by enemies, namely managed under the influence of individuals or corporations themselves considered as enemies... The Philippine Insurance Law (Act No 2427, as amended), in Section 8, provides that "anyone except a public enemy may be insured". It stands to reason that an insurance policy ceases to be allowable as soon as an insured becomes a public enemy. The respondent having an enemy corporation on December 10, 1941, the insurance policy issued in its favor on October 1, 1941, by the petitioner had ceased to be valid and enforceable, and since the insured good were burned during the war, the respondent was not entitled to any indemnity under said policy from the petitioner. However, elementary rule of justice (in the absence of specific provisions in the Insurance Law) require that the premium paid by the respondent for the period covered by its policy from December 11, 1941, should be returned by the petitioner. San Miguel Brewery v. Law Union Rock Insurance Company - Insurance Proceeds 40 PHIL 674 Facts: > On Jan. 12, 1918, Dunn mortgaged a parcel of land to SMB to secure a debt of 10T. > Mortgage contract stated that Dunn was to have the property insured at his own expense, authorizing SMB to choose the insurers and to receive the proceeds thereof and retain so much of the proceeds as would cover the mortgage debt. > Dunn likewise authorized SMB to take out the insurance policy for him. > Brias, SMBs general manager, approached Law Union for insurance to the extent of 15T upon the property. In the application, Brias stated that SMBs interest in the property was merely that of a mortgagee. > Law Union, not wanting to issue a policy for the entire amount, issued one for P7,500 and procured another policy of equal amount from Filipinas Cia de Seguros. Both policies were issued in the name of SMB only and contained no reference to any other interests in the propty. Both policies required assignments to be approved and noted on the policy. > Premiums were paid by SMB and charged to Dunn. A year later, the policies were renewed. > In 1917, Dunn sold the property to Harding, but no assignment of the policies was made to the latter. > Property was destroyed by fire. SMB filed an action in court to recover on the policies. Harding was made a defendant because by virtue of the sale, he became the owner of the property, although the policies were issued in SMBs name. > SMB sought to recover the proceeds to the extent of its mortgage credit with the balance to go to Harding. > Insurance Companies contended that they were not liable to Harding because their liability under the policies was limited to the insurable interests of SMB only. > SMB eventually reached a settlement with the insurance companies and was paid the balance of its mortgage credit. Harding was left to fend for himself. Trial court ruled against Harding. Hence the appeal. Issue: Whether or not the insurance companies are liable to Harding for the balance of the proceeds of the 2 policies.

Held: NOPE. Under the Insurance Act, the measure of insurable interest in the property is the extent to which the insured might be daminified by the loss or injury thereof. Also it is provided in the IA that the insurance shall be applied exclusively to the proper interest of the person in whose name it is made. Undoubtedly, SMB as the mortgagee of the property, had an insurable interest therein; but it could NOT, an any event, recover upon the two policies an amount in excess of its mortgage credit. By virtue of the Insurance Act, neither Dunn nor Harding could have recovered from the two policies. With respect to Harding, when he acquired the property, no change or assignment of the policies had been undertaken. The policies might have been worded differently so as to protect the owner, but this was not done. Saura Import Export Co. v. Philippine International Surety - Cancellation of Policy 118 PHIL 150 Facts: > On Dec. 26, 1952, Saura mortgaged to PNB its registered parcel of land in Davao to secure the payment of a promissory note of P27T. > A building of strong materials which was also owned by Saura, was erected on the parcel of land and the building had always been covered by insurance even before the execution of the mortgage contract. > Pursuant to the mortgage agreement which required Saura to insure the building and its contents, it obtained a fire insurance for P29T from PISC for a period of 1 year starting Oct. 2, 1954. > The mortgage also required Saura to endorse the insurance policy to PNB. The memo stated: Loss if any, payable to PNG as their interest may appear, subject to the terms, conditions and warranties of this policy. > The policy was delivered to PNB by Saura. > On Oct. 15, 1954, barely 13 days after the issuance of the fire insurance, PISC canceled the same, effective as of the date of issue. Notice of the cancellation was sent to PNB in writing and was received by the bank on Nov. 8, 1954. > On Apr. 6, 1955, the building and its contents worth P4,685 were burned. On April 11, 1985, Saura filed a claim with PISC and mortgagee bank. > Upon presentation of notice of loss with PNB, Saura learned for the first time that the policy had been previously canceled by PISC, when Sauras folder in the banks file was opened and the notice of the cancellation by PISC was found. Issue: Whether or not there was proper cancellation of the policy? Held: NO. The policy in question does NOT provide for the notice of cancellation, its form or period. The Insurance Law does not likewise provide for such notice. This being the case, it devolves upon the Court to apply the generally accepted principles of insurance, regarding cancellation of the insurance policy by the insurer. Actual notice of cancellation in a clear and unequivocal manner, preferably in writing should be given by the insurer to the insured so that the latter might be given an opportunity to obtain other insurance for his own protection. The notice should be personal to the insurer and not to and/or through any unauthorized person by the policy. Both the PSIC and the PNB failed, wittingly or unwittingly to notify Saura of the cancellation made. The insurer contends that it gave notice to PNB as mortgagee of the property and that was already substantial compliance with its duty to notify the insured of the cancellation of the policy. But notice to the bank, as far as Saura herein is concerned, is not effective notice. PISC is then ordered to pay Saura P29T, the amount involved in the policy subject matter of this case. Grepalife v. CA - Real Party In Interest

316 SCRA 677 Facts: > A contract of group life insurance was executed between Grepalife and DBP. Grepalife agreed to insure the lives of eligible housing loan mortgagors of DBP. > Dr. Wilfredo Leuterio, a physician and a housing debtor of DBP applied for membership in the group life insurance plan. > In an application form, Dr. Leuterio answered questions concerning his health stating that he is in good health and has never consulted a physician for or a heart condition, high blood pressure, cancer, diabetes, lung, kidney or stomach disorder or any other physical impairment. > Grepalife issued the insurance coverage of Dr. Leuterio, to the extent of his DBP mortgage indebtedness amounting to eighty-six thousand, two hundred (P86,200.00) pesos. > Dr. Leuterio died due to "massive cerebral hemorrhage." Consequently, DBP submitted a death claim to Grepalife. > Grepalife denied the claim alleging that Dr. Leuterio was not physically healthy when he applied for an insurance coverage and insisted that Dr. Leuterio did not disclose that he had been suffering from hypertension, which caused his death. Allegedly, such non-disclosure constituted concealment that justified the denial of the claim. > The widow of the late Dr. Leuterio, filed a complaint against Grepalife for "Specific Performance with Damages." During the trial, Dr. Hernando Mejia, who issued the death certificate, was called to testify. Dr. Mejias findings, based partly from the information given by the widow, stated that Dr. Leuterio complained of headaches presumably due to high blood pressure. The inference was not conclusive because Dr. Leuterio was not autopsied, hence, other causes were not ruled out. > RTC ruled in favor of widow and against Grepalife. Grepalife appealed contending that the wife was not the proper party in interest to file the suit, since it is DBP who insured the life of Dr. Leuterio. Issue: Whether or not the widow is the real party in interest, (not DBP) and has legal standing to file the suit. Held: YES. Grepalife alleges that the complaint was instituted by the widow of Dr. Leuterio, not the real party in interest, hence the trial court acquired no jurisdiction over the case. It argues that when the Court of Appeals affirmed the trial courts judgment, Grepalife was held liable to pay the proceeds of insurance contract in favor of DBP, the indispensable party who was not joined in the suit. To resolve the issue, we must consider the insurable interest in mortgaged properties and the parties to this type of contract. The rationale of a group insurance policy of mortgagors, otherwise known as the "mortgage redemption insurance," is a device for the protection of both the mortgagee and the mortgagor. On the part of the mortgagee, it has to enter into such form of contract so that in the event of the unexpected demise of the mortgagor during the subsistence of the mortgage contract, the proceeds from such insurance will be applied to the payment of the mortgage debt, thereby relieving the heirs of the mortgagor from paying the obligation. In a similar vein, ample protection is given to the mortgagor under such a concept so that in the event of death; the mortgage obligation will be extinguished by the application of the insurance proceeds to the mortgage indebtedness. Consequently, where the mortgagor pays the insurance premium under the group insurance policy, making the loss payable to the mortgagee, the insurance is on the mortgagors interest, and the mortgagor continues to be a party to the contract. In this type of policy insurance, the mortgagee is simply an appointee of the insurance fund, such losspayable clause does not make the mortgagee a party to the contract. The insured private respondent did not cede to the mortgagee all his rights or interests in the insurance, the policy stating that: "In the event of the debtors death before his indebtedness with the Creditor [DBP] shall have been fully paid, an amount to pay the outstanding indebtedness shall first be paid to the creditor and the balance of sum assured, if there is any, shall then be paid to the beneficiary/ies designated by the debtor." When DBP submitted the insurance claim against petitioner, the latter denied payment thereof, interposing the defense of concealment committed by the insured. Thereafter, DBP collected the debt from the mortgagor and took the necessary action of foreclosure on the residential lot of private respondent

And since a policy of insurance upon life or health may pass by transfer, will or succession to any person, whether he has an insurable interest or not, and such person may recover it whatever the insured might have recovered, 14 the widow of the decedent Dr. Leuterio may file the suit against the insurer, Grepalife. As to the question of whether there was concealment, CA held as affirmed by the SC that contrary to Grepalifes allegations, there was no sufficient proof that the insured had suffered from hypertension. Aside from the statement of the insureds widow who was not even sure if the medicines taken by Dr. Leuterio were for hypertension, the appellant had not proven nor produced any witness who could attest to Dr. Leuterios medical history. The fraudulent intent on the part of the insured must be established to entitle the insurer to rescind the contract. Misrepresentation as a defense of the insurer to avoid liability is an affirmative defense and the duty to establish such defense by satisfactory and convincing evidence rests upon the insurer. In the case at bar, the petitioner failed to clearly and satisfactorily establish its defense, and is therefore liable to pay the proceeds of the insurance Gercio v. Sun Life - Insurance Beneficiary 48 PHIL 53 Facts: > Sunlife issued a life insurance policy to Gercio, the former agreeing to insure the life of Gercio for 2T to be paid to him on Feb. 1, 1930 or if he should die before said date, then to his wife Andrea, should she survive him; otherwise to the executor, administrator of Gercio. > The policy did not include any provision reserving to Gercio the right to change the beneficiary. > The wife was convicted of adultery and a decree of divorce was issued. > Gercio notified Sunlife that he had revoked his donation in favor of Andrea and that he had designated his present wife Adela as his beneficiary. > Sunlife refused to change the beneficiary. Issue: Whether or not Gercio may change the beneficiary in the policy. Held: NO. If the policy contains no provision authorizing a change of beneficiary without the beneficiarys consent, the insured cannot make such change. It is held that a life insurance policy of a husband made payable to his wife as a beneficiary is the separate property of the beneficiary and beyond the control of the husband. (NOTE: this case is based on the old rule under the Insurance Act) Court also held that the designation of a beneficiary that is originally valid does NOT render it invalid dut to a subsequent cessation of the interests between the beneficiary and insured. Philamcare vs. ca Facts: > Ernani Trinos, applied for a health care coverage with Philamcare. In the standard application form, he answered NO to the following question: Have you or any of your family members ever consulted or been treated for high blood pressure, heart trouble, diabetes, cancer, liver disease, asthma or peptic ulcer? (If Yes, give details) > The application was approved for a period of one year from March 1, 1988 to March 1, 1989. He was a issued Health Care Agreement, and under such, he was entitled to avail of hospitalization benefits, whether ordinary or emergency, listed therein. He was also entitled to avail of "out-patient benefits" such as annual physical examinations, preventive health care and other out-patient services. > Upon the termination of the agreement, the same was extended for another year from March 1, 1989 to March 1, 1990, then from March 1, 1990 to June 1, 1990. The amount of coverage was increased to a maximum sum of P75,000.00 per disability. > During the period of his coverage, Ernani suffered a heart attack and was confined at the Manila Medical Center (MMC) for one month beginning March 9, 1990. > While her husband was in the hospital, Julita tried to claim the benefits under the health care agreement. However, Philamcare denied her claim saying that the Health Care Agreement was void. > According to Philamcare, there was concealment regarding Ernani's medical history.

Doctors at the MMC allegedly discovered at the time of Ernani's confinement that he was hypertensive, diabetic and asthmatic, contrary to his answer in the application form. > Julita had no choice but to pay the hospitalization expenses herself, amounting to about P76,000.00 > After her husband was discharged from the MMC, he was attended by a physical therapist at home. Later, he was admitted at the Chinese General Hospital (CGH). Due to financial difficulties, Julita brought her husband home again. In the morning of April 13, 1990, Ernani had fever and was feeling very weak. Julita was constrained to bring him back to the CGH where he died on the same day. > Julita instituted, an action for damages against Philamcare. She asked for reimbursement of her expenses plus moral damages and attorney's fees. RTC decided in favor of Julita. Ca affirmed. Issues and Resolutions: Philamcare brought the instant petition for review, raising the primary argument that a health care agreement is not an insurance contract; hence the "incontestability clause" under the Insurance Code Title 6, Sec. 48 does not apply. SC held that in the case at bar, the insurable interest of respondent's husband in obtaining the health care agreement was his own health. The health care agreement was in the nature of non-life insurance, which is primarily a contract of indemnity. Once the member incurs hospital, medical or any other expense arising from sickness, injury or other stipulated contingent, the health care provider must pay for the same to the extent agreed upon under the contract. Under the title Claim procedures of expenses, Philamcare. had 12 mos from the date of issuance of the Agreement within which to contest the membership of the patient if he had previous ailment of asthma, and six months from the issuance of the agreement if the patient was sick of diabetes or hypertension. The periods having expired, the defense of concealment or misrepresentation no longer lie. Petitioner argues that respondent's husband concealed a material fact in his application. It appears that in the application for health coverage, petitioners required respondent's husband to sign an express authorization for any person, organization or entity that has any record or knowledge of his health to furnish any and all information relative to any hospitalization, consultation, treatment or any other medical advice or examination. Philamcare cannot rely on the stipulation regarding "Invalidation of agreement" which reads: Failure to disclose or misrepresentation of any material information by the member in the application or medical examination, whether intentional or unintentional, shall automatically invalidate the Agreement from the very beginning and liability of Philamcare shall be limited to return of all Membership Fees paid. An undisclosed or misrepresented information is deemed material if its revelation would have resulted in the declination of the applicant by Philamcare or the assessment of a higher Membership Fee for the benefit or benefits applied for. The answer assailed by petitioner was in response to the question relating to the medical history of the applicant. This largely depends on opinion rather than fact, especially coming from respondent's husband who was not a medical doctor. Where matters of opinion or judgment are called for, answers made in good faith and without intent to deceive will not avoid a policy even though they are untrue. Thus, (A)lthough false, a representation of the expectation, intention, belief, opinion, or judgment of the insured will not avoid the policy if there is no actual fraud in inducing the acceptance of the risk, or its acceptance at a lower rate of premium, and this is likewise the rule although the statement is material to the risk, if the statement is obviously of the foregoing character, since in such case the insurer is not justified in relying upon such statement, but is obligated to make further inquiry. There is a clear distinction between such a case and one in which the insured is fraudulently and intentionally states to be true, as a matter of expectation or belief, that which he then knows, to be actually untrue, or the impossibility of which is shown by the facts within his knowledge, since in such case the intent to deceive the insurer is obvious and amounts to actual fraud. The fraudulent intent on the part of the insured must be established to warrant rescission of the insurance contract. Concealment as a defense for the health care provider or insurer to avoid liability is an affirmative defense and the duty to establish such defense by satisfactory and convincing evidence rests upon the provider or insurer. In any case, with or without the authority to investigate, petitioner is liable for claims made under the contract. Having assumed a responsibility under the agreement, petitioner is bound to answer the same to the extent agreed upon. In the end, the liability of the health care provider attaches once the member is hospitalized for the disease or injury covered by the agreement or whenever he avails of the covered benefits which he has prepaid. Under Section 27 of the Insurance Code, "a concealment entitles the injured party to rescind a contract of insurance." The right to rescind should be exercised previous to the commencement of an action on the contract. In

this case, no rescission was made. Besides, the cancellation of health care agreements as in insurance policies require the concurrence of the following conditions: 1. Prior notice of cancellation to insured; 2. Notice must be based on the occurrence after effective date of the policy of one or more of the grounds mentioned; 3. Must be in writing, mailed or delivered to the insured at the address shown in the policy; 4. Must state the grounds relied upon provided in Section 64 of the Insurance Code and upon request of insured, to furnish facts on which cancellation is based. None of the above pre-conditions was fulfilled in this case. When the terms of insurance contract contain limitations on liability, courts should construe them in such a way as to preclude the insurer from non-compliance with his obligation. Being a contract of adhesion, the terms of an insurance contract are to be construed strictly against the party which prepared the contract the insurer. By reason of the exclusive control of the insurance company over the terms and phraseology of the insurance contract, ambiguity must be strictly interpreted against the insurer and liberally in favor of the insured, especially to avoid forfeiture. This is equally applicable to Health Care Agreements. The phraseology used in medical or hospital service contracts, such as the one at bar, must be liberally construed in favor of the subscriber, and if doubtful or reasonably susceptible of two interpretations the construction conferring coverage is to be adopted, and exclusionary clauses of doubtful import should be strictly construed against the provider. If the wording had been: Payable to SMB, mortgagee, as its interests may appear, remainder to whomsoever, during the continuance of the risk, may become owner of the interest insured , it would have proved an intention to insure the entire interest in the property, NOT merely SMBs and would have shown to whom the money, in case of loss, should be paid. Unfortunately, this was not what was stated in the policies. If during the negotiation for the policies, the parties had agreed that even the owners interest would be covered by the policies, and the policies had inadvertently been written in the form in which they were eventually issued, the lower court would have been able to order that the contract be reformed to give effect to them in the sense that the parties intended to be bound. However, there is no clear and satisfactory proof that the policies failed to reflect the real agreement between the parties that would justify the reformation of these two contracts. Insular Life vs. Ebrado 80 SCRA 181 Facts: > Buenaventura Ebrado was issued al life plan by Insular Company. He designated Capriona as his beneficiary, referring to her as his wife. > The insured then died and Carponia tried to claim the proceeds of the said plan. > She admitted to being only the common law wife of the insured. > Pascuala, the legal wife, also filed a claim asserting her right as the legal wife. The company then filed an action for interpleader. Issue: Whether or not the common law wife named as beneficiary can collect the proceeds. Held: NO. The civil code prohibitions on donations made between persons guilty of adulterous concubinage applies to insurance contracts. On matters not specifically provided for by the Insurance Law, the general rules on Civil law shall apply. A life insurance policy is no different from a civil donation as far as the beneficiary is concerned, since both are founded on liberality. Why was the common law wife not ed to collect the proceeds despite the fact that she was the beneficiary? Isnt this against Sec. 53? It is true that SC went against Sec. 53. However, Sec. 53 is NOT the only provision that the SC had to consider. Art. 739 and 2012 of CC prohibit persons who are guilty of adultery or concubinage from being

beneficiaries of the life insurance policies of the persons with whom they committed adultery or concubinage. If the SC used only Sec. 53, it would have gone against Art. 739 and 2012. Philamlife v. Pineda - Life Insurance 175 SCRA 416 Facts: > On Jan. 15 1963, Dimayuga processed an ordinary life insurance policy from Philamlife and designated his wife and children as irrevocable beneficiaries. > On Feb. 22, 1980, Dimayuga filed a petition in court to amend the designation of the beneficiaries in his policy from irrevocable to revocable. > Lower Court granted the petition. Issue: Whether or not the court erred in granting Dimayugas petition. Held: YES. Under the Insurance Act, the beneficiary designated in a life insurance contract cannot be changed without the consent of the beneficiary because he has a vested interest in the policy. The policy contract states that the designation of the beneficiaries is irrevocable. Therefore, based on the said provision of the contract, not to mention the law then applicable, it is only with the consent of all the beneficiaries that any change or amendment in the poicy may be legally and validly effected. The contract between the parties is the law binding on them. (This case rule is no longer controlling under the Insurance Code.) Vicente Ong Lim Sing, Jr. v. Feb Leasing and Finance Corp. GR no. 168115 June 8, 2007 Nachura, J. FACTS FEB Leasing and Finance Corp entered into a lease agreement of equipment and motor vehicles with JVL Food Products. Vicente Ong Lim Sing, Jr. executed an Individual Guarantee Agreement with FEB regarding faithful compliance with the terms of the lease agreement. JVL defaulted on its obligation. By 2000, the arrears of JVL amounted to P3,414,468.75. Due to the continuous nonpayment despite numerous demands, FEB filed a complaint for sum of money, damages, and replevin against JVL and Lim. JVL and Lim argued before the court that the lease contract was actually a sale on installment basis. They further argued that the contract was a contract of adhesion. The trial court rendered a ruling In favor of Lim and JVL. The trial court, through logic, ruled that Lim cannot be a mere lessee because of he had an insurable interest over the items. It has also been held that the test of insurable interest in property is whether the assured has a right, title or interest therein that he will be benefited by its preservation and continued existence or suffer a direct pecuniary loss from its destruction or injury by the peril insured against. If Lim and JVL were to be regarded as only a lessee, logically the lessor who asserts ownership will be the one directly benefited or injured and therefore the lessee is not supposed to be the assured as he has no insurable interest. FEB appealed the decision before the Court of Appeals. The appellate court rendered judgment in favor of FEB. It reversed the earlier decision of the RTC of Manila and ordered Lim and JVL to pay FEB the amount due plus damages. Unsatisfied with the decision, JVL and Lim appealed the case before the Supreme Court. ISSUE Whether or not a lease agreement was executed by JVL and FEB.

RULING The Supreme Court dismissed the petition of Lim and affirmed the decision of the Court of Appeals. According to the Court, the agreement was indeed a financial lease agreement and not a sale by installment basis. The Court also ruled that the lessee, herein petitioner, had an insurable interest in the items even if he was only a lessee. Section 17 of the Insurance Code provides that the measure of an insurable interest in property is the extent to which the insured might be damnified by loss or injury thereof. It cannot be denied that JVL will be directly damnified in case of loss, damage, or destruction of any of the properties leased. Traders Insurance and Surety Co. v. Golangco- Insurance Proceeds 95 PHIL 826 Facts: > A decision was rendred in Civil Case No. 6306 granting Golangco the right to collect rentals from a building in Sta. Cruz, Manila. > Golangco then sought fire insurance from Traders. Before the policy was issued, Golangco made a full and clear exposal of his interests in the premises, i.e. that he was not the owner. > The fire policy that defendant issued covered only all of Golangcos intere st in the premises and his right to collect the rentals. > The building burned down in a fire and Golangco sought to collect from Traders. Traders denied any liability on the ground that since Golangco was not the owner of the premises then he had no insurable interest in the same and consequently, he could not collect the insurance proceeds. Issue: Whether or not plaintiff can claim the insurance proceeds. Held. YES. Both at the time of the issuance of the policy and at the time of the fire, plaintiff Golangco was in legal possession of the premises, collecting rentals from its occupant. It seems plain that if the premises were destroyed as they were, by fire, Golangco would be, as he was, directly damnified thereby; and hence he had an insurable interest therein. Lampano v. Jose (1915) FACTS: Mariano R. Barretto, constructed a house for Placida A. Jose sold the house to Antonina Lampano for P6,000 The house was destroyed by fire during which Lampano still owed Jose P2,000 as evidenced by a promissory note. Jose also owed Barretto P2,000 for the construction. After the completion of the house and before it was destroyed, Mariano R. Barretto took out an insurance policy upon it in his own name, with the consent of Placida A. Jose, for the sum of P4,000. After its destruction, he collected P3,600 from the insurance company, having paid in premiums the sum of P301.50 Lampano filed a complaint against Barreto and Jose alleging that Jose in a verbal agreement told her that the policy will be delivered to her so she should collected P3,600 from each of them RTC: favored Jose ordering Barreto to pay him P1,298.50 and offsetting the P2,000 Barreto alone appealed ISSUE: W/N Barreto had insurable interest in the house and could insure it for his it for his own protection HELD: YES. reversed and Barretto is absolved Where different persons have different interests in the same property, the insurance taken by one in his own right and in his own interest does not in any way insure to the benefit of another A contract of insurance made for the insurer's (insured) indemnity only, as where there is no agreement, express or implied, that it shall be for the benefit of a third person, does not attach to or run with the title to the insured property on a transfer thereof personal as between the insurer and the insured. Barretto had an insurable interest in the house. He construed the building, furnishing all the materials and supplies, and insured it after it had been completed

Lopez v. Del Rosario 44 PHIL 98 Facts: > Benita Del Rosario is the owner of a bonded warehouse in Manila where copra and other merchandise are deposited. > Among those who had copra deposited in the warehouse was Froilan Lopez, the owner of 14 warehouse receipts with a declared value of P107,990.40 in his name. > Del Rosario secured insurance on the warehouse and its contents with 5 different insurance companies in the amount of P404,800. > All policies were in the name of Del Rosario, except for one (with Natl Insurance Co.) for 40T, in favor of Compania Copra de Tayabas. > The warehouse and its contents were destroyed by fire. When Bayne, a fire loss adjuster, failed to effect a settlement between the Insurance companies and Del Rosario, the latter authorized Atty. Fisher to negotiate with the Companies. > An agreement was reached to submit the matter to arbitration. The claims by different people who had stored copra in the warehouse were settled with the exception of Friolan Lopez. > A case was filed in CFI by Lopez. The court awarded him the sum of P88,492.21 with legal interest. Issue: Whether or not Del Rosario acted as the agent of Lopez in taking out the insurance on the contents of the warehouse or whether she acted as the reinsurer of the copra. Held: She acted as the agent of Lopez. The agency can be deduced from the warehouse receipts, the insurance policies and the circumstances surrounding the transaction. Under any aspect, Del Rosario is liable. The law is that a policy effected by a bailee and covering by its terms in his own property and property held in trust, inures, in the event of loss, equally and proportionately to the benefit of all owners of the property insured. Even if one secured insurance covering his own goods and goods stored with him, and even if the owner of the stored goods did not request or know the insurance, and did not ratify it before the payment of the loss, it has been held by a reputable court that the warehouseman is liable to the owner of such stored goods for his share. In a case of contributing policies, adjustments of loss made by an expert or by a board of arbitrators may be submitted to the court NOT as evidence of the facts stated therein, or as obligatory, but for the purpose of assisting the court in calculating the amount of liability. Sharuff & Co. v. Baloise Fire Insurance Co. (1937) FACTS: Salomon Sharruf and Elias Eskenazi were doing business under the firm name of Sharruf & Co. They insured their stocks with aloise Fire Insurance Co., Sun Insurance Office Ltd., and Springfield Insurance Co. raising it to P40,000. Elias Eskenazi having paid the corresponding premiums Soon they changed the name of their partnership to Sharruf & Eskenazi September 22, 1933: A fire ensued at their building at Muelle de la Industria street where petroleum was spilt lasting 27 minutes Sharruf & Co. claimed 40 cases when only 10 or 11 partly burned and scorched cases were found RTC: ordered Baloise Fire Insurance Co., Sun Insurance Office Ltd., and Springfield Insurance Co., to pay the partners Salomon Sharruf and Elias Eskenazi P40,000 plus 8% interest ISSUE: W/N Sharruf & Eskenazi has juridical personality and insurable interest HELD: YES. Reversd. Insurance companies are absolved. It does not appear that in changing the title of the partnership they had the intention of defrauding the insurance companies

fire which broke out in the building at Nos. 299-301 Muelle de la Industria, occupied by Sharruf & Eskenazi but no evidence sufficient to warrant a finding that they are responsible for the fire So great is the difference between the amount of articles insured, which the plaintiffs claim to have been in the building before the fire, and the amount thereof shown by the vestige of the fire to have been therein, that the most liberal human judgment can not attribute such difference to a mere innocent error in estimate or counting but to a deliberate intent to demand of the insurance companies payment of an indemnity for goods not existing at the time of the fire, thereby constituting the so-called "fraudulent claim" which, by express agreement between the insurers and the insured, is a ground for exemption of the insurers from civil liability acted in bad faith in presenting a fraudulent claim, they are not entitled to the indemnity claimed when the partners of a general partnership doing business under the firm name of "Sharruf & Co." obtain insurance policies issued to said firm and the latter is afterwards changed to "Sharruf & Eskenazi", which are the names of the same and only partners of said firm "Sharruf & Co.", continuing the same business, the new firm acquires the rights of the former under the same policies; Cha v. Cha - Insurable Interest 277 SCRA 690 (1997) Facts: > Spouses Nilo Cha and Stella Uy-Cha, as lessees, entered into a lease contract with CKS Development Corporation (CKS), as lessor. > One of the stipulations of the one (1) year lease contract states: "18. . . . The LESSEE shall not insure against fire the chattels, merchandise, textiles, goods and effects placed at any stall or store or space in the leased premises without first obtaining the written consent and approval of the LESSOR. If the LESSEE obtain(s) the insurance thereof without the consent of the LESSOR then the policy is deemed assigned and transferred to the LESSOR for its own benefit; . . ." > Notwithstanding the above stipulation, the Cha spouses insured against loss by fire their merchandise inside the leased premises for Five Hundred Thousand (P500,000.00) with the United Insurance without the written consent CKS. > On the day that the lease contract was to expire, fire broke out inside the leased premises. When CKS learned of the insurance earlier procured by the Cha spouses (without its consent), it wrote the United a demand letter asking that the proceeds of the insurance contract (between the Cha spouses and United) be paid directly to CKS, based on its lease contract with the Cha spouses. > United refused to pay CKS, alleging that the latter had no insurable interest. Hence, the latter filed a complaint against the Cha spouses and United. Issue: Whether or not CKS can claim the proceeds of the fire insurance. Held: NO. CKS has no insurable interest. Sec. 18 of the Insurance Code provides: "Sec. 18. No contract or policy of insurance on property shall be enforceable except for the benefit of some person having an insurable interest in the property insured." A non-life insurance policy such as the fire insurance policy taken by petitioner-spouses over their merchandise is primarily a contract of indemnity. Insurable interest in the property insured must exist at the time the insurance takes effect and at the time the loss occurs. The basis of such requirement of insurable interest in property insured is based on sound public policy: to prevent a person from taking out an insurance policy on property upon which he has no insurable interest and collecting the proceeds of said policy in case of loss of the property. In the present case, it cannot be denied that CKS has no insurable interest in the goods and merchandise inside the leased premises under the provisions of Section 17 of the Insurance Code which provide: "Section 17. The measure of an insurable interest in property is the extent to which the insured might be damnified by loss of injury thereof." Therefore, CKS cannot, under the Insurance Code a special law be validly a beneficiary of the fire insurance policy taken by the petitioner-spouses over their merchandise. This insurable interest over said merchandise remains with the insured, the Cha spouses. The automatic assignment of the policy to CKS under the provision of the lease contract previously quoted is void for being contrary to law and/or public policy. The proceeds of the fire insurance

policy thus rightfully belong to the spouses Nilo Cha and Stella Uy-Cha (herein co-petitioners). The insurer (United) cannot be compelled to pay the proceeds of the fire insurance policy to a person (CKS) who has no insurable interest in the property insured. Tai Tong v Insurance G.R. No. L-55397 February 29, 1988 J. Gancayco Facts: Azucena Palomo obtained a loan from Tai Tong Chuache Inc. in the amount of P100,000.00. To secure the payment of the loan, a mortgage was executed over the land and the building in favor of Tai Tong Chuache & Co. Arsenio Chua, representative of Thai Tong Chuache & Co. insured the latter's interest with Travellers Multi-Indemnity Corporation for P100,000.00 (P70,000.00 for the building and P30,000.00 for the contents thereof) Pedro Palomo secured a Fire Insurance Policy covering the building for P50,000.00 with respondent Zenith Insurance Corporation. On July 16, 1975, another Fire Insurance was procured from respondent Philippine British Assurance Company, covering the same building for P50,000.00 and the contents thereof for P70,000.00. The building and the contents were totally razed by fire. Based on the computation of the loss, including the Travellers Multi- Indemnity, respondents, Zenith Insurance, Phil. British Assurance and S.S.S. Accredited Group of Insurers, paid their corresponding shares of the loss. Complainants were paid the following: P41,546.79 by Philippine British Assurance Co., P11,877.14 by Zenith Insurance Corporation, and P5,936.57 by S.S.S. Group of Accredited Insurers Demand was made from respondent Travellers Multi-Indemnity for its share in the loss but the same was refused. Hence, complainants demanded from the other three (3) respondents the balance of each share in the loss in the amount of P30,894.31 (P5,732.79-Zenith Insurance: P22,294.62, Phil. British: and P2,866.90, SSS Accredited) but the same was refused, hence, this action. In their answers, Philippine British Assurance and Zenith Insurance Corporation denied liability on the ground that the claim of the complainants had already been waived, extinguished or paid. Both companies set up counterclaim in the total amount of P 91,546.79. SSS Accredited Group of Insurers informed the Commission that the claim of complainants for the balance had been paid in the amount in full. Travellers Insurance, on its part, admitted the issuance of a Policy and alleged defenses that Fire Policy, covering the furniture and building of complainants was secured by a certain Arsenio Chua and that the premium due on the fire policy was paid by Arsenio Chua. Tai Tong Chuache & Co. also filed a complaint in intervention claiming the proceeds of the fire Insurance Policy issued by respondent Travellers Multi-Indemnity. As adverted to above respondent Insurance Commission dismissed spouses Palomos' complaint on the ground that the insurance policy subject of the complaint was taken out by Tai Tong Chuache & Company, for its own interest only as mortgagee of the insured property and thus complainant as mortgagors of the insured property have no right of action against the respondent. It likewise dismissed petitioner's complaint in intervention in the following words: From the above decision, only intervenor Tai Tong Chuache filed a motion for reconsideration but it was likewise denied hence, the present petition. Issue: WON Tai Tong had insurable interest Held: Yes. Petition granted. Ratio: Respondent advanced an affirmative defense of lack of insurable interest on the part of the petitioner that before the occurrence of the peril insured against, the Palomos had already paid their credit due the petitioner. However, they were never able to prove that Tai had a lack of insurable interest. Hence, the decision must be adverse against them. However respondent Insurance Commission absolved respondent insurance company from liability on the basis of the certification issued by the then Court of First Instance of Davao, Branch II, that in a certain civil action against the Palomos, Arsenio Lopez Chua stands as the complainant and not Tai Tong Chuache. From said evidence respondent commission inferred that the credit extended by petitioner to the Palomos secured by the insured property must have been paid. These findings was based upon a mere inference. The record of the case shows that the petitioner to support its claim for the insurance proceeds offered as evidence the contract of mortgage which has not been cancelled nor released. It has been held in a long line of cases that when the creditor is in possession of the document of credit, he need not prove non-payment for it is presumed. The validity of the insurance policy taken by petitioner was not assailed by private respondent. Moreover, petitioner's

claim that the loan extended to the Palomos has not yet been paid was corroborated by Azucena Palomo who testified that they are still indebted to herein petitioner. Public respondent argues however, that if the civil case really stemmed from the loan granted to Azucena Palomo by petitioner the same should have been brought by Tai Tong Chuache or by its representative in its own behalf. From the above premise, respondent concluded that the obligation secured by the insured property must have been paid. However, it should be borne in mind that petitioner being a partnership may sue and be sued in its name or by its duly authorized representative. Petitioner's declaration that Arsenio Lopez Chua acts as the managing partner of the partnership was corroborated by respondent insurance company. Thus Chua as the managing partner of the partnership may execute all acts of administration including the right to sue debtors of the partnership in case of their failure to pay their obligations when it became due and demandable. Public respondent's allegation that the civil case flied by Arsenio Chua was in his capacity as personal creditor of spouses Palomo has no basis. The policy, then had legal force and effect. Bachrach v. British American Insurance Co. - Insurance Proceeds 17 PHIL 555 Facts: > Bachrach insured properties of its general furniture shop with British. The properties were subsequently destroyed by fire. > Bachrach claims from the insurance company. The claim was denied on the ff grounds: o The policy was allegedly forfeited because the insured stored varnishes and paints within the premises; o Insured stored gasoline in the building; and o Bachrach executed a chattel mortgage on the properties insured without the consent of the insured. Issue: Whether or not Bachrach can claim the proceeds of the policy. Held: Yes. The policy was NOT forfeited due to the strong paints and varnishes. There was no express provision pertaining to it and these paints and varnishes are incidental to the business of the insured to keep the furniture in a saleable condition. The gasoline stored within the premises was in the reservoir of the car and thus does not violate any provision in the policy. There is no express prohibition against the execution of a chattel mortgage on the property insured. Yu Pang Cheng v. CA- Life Insurance Policy 105 PHIL 1930 Facts: > Yu Pang Eng obtained a life insurance policy naming his brother Yu Pang Cheng as beneficiary. > Eng subsequently died of medullary carcinoma, Grade 4, advanced and lesser curvature. > Cheng claims the proceeds of the policy. > Insurance co. refused payment on the ground that the policy was void due to the concealment. Issue: Whether or not the policy is void. Held: YES. In the application for the policy, Eng was asked whether he had been ill or had consulted a doctor due to symptoms or illnesses enumerated in the questionnaire. He answered No, when in fact he was hospitalized seven months prior to his application for the said policy. Grepalife v. CA 89 SCRA 543 Facts: > On March 14, 1957, respondent Ngo Hing filed an application with Grepalife for a 20-yr endowment policy for 50T on the life of his one year old daughter Helen Go. > All the essential data regarding Helen was supplied by Ngo to Lapu-Lapu Mondragon, the branch manager of Grepalife-Cebu. Mondragon then typed the data on the application form which was later signed by Ngo.

> Ngo then paid the insurance premium and a binding deposit receipt was issued to him. The binding receipt contained the following provision: If the applicant shall not have been insurable xxx and the Company declines to approve the application, the insurance applied for shall not have been in force at any time and the sum paid shall be returned to the applicant upon the surrender of this receipt. > Mondragon wrote on the bottom of the application form his strong recommendation for the approval of the insurance application. > On Apr 30, 1957, Mondragon received a letter from Grepalife Main office disapproving the insurance application of Ngo for the simple reason that the 20yr endowment plan is not available for minors below 7 yrs old. > Mondragon wrote back the main office again strongly recommending the approval of the endowment plan on the life of Helen, adding that Grepalife was the only insurance company NOT selling endowment plans to children. > On may 1957, Helen died of influenza with complication of broncho pneumonia. Ngo filed a claim with Gepalife, but the latter denied liability on the ground that there was no contract between the insurer and the insured and a binding receipt is NOT evidence of such contract. Issue: Whether or not the binding deposit receipt, constituted a temporary contract of life insurance. Held: NO. The binding receipt in question was merely an acknowledgement on behalf of the company, that the latters branch office had received from the applicant, the insurance premium and had accepted the application subject for processing by the insurance company, and that the latter will either approve or reject the same on the basis of whether or not the applicant is insurable on standard rates. Since Grepalife disapproved the insurance application of Ngo, the binding deposit receipt had never became on force at any time, pursuant to par. E of the said receipt. A binding receipt is manifestly merely conditional and does NOT insure outright. Where an agreement is made between the applicant and the agent, NO liability shall attach until the principal approves the risk and a receipt is given by the agent. The acceptance is merely conditional, and is subordinated to the act of the company in approving or rejecting the application. Thus in life insurance, a binding slip or binding receipt does NOT insure by itself. Gen. Insurance & Surety Corp v. NG Hua - Misrepresentation 106 PHIL 1117 Facts: > In 1952, General issued a fire policy to Ng Hua to cover the contents of the Central Pomade Factory owned by him. > There was a provision in the policy that should there be any insurance already effected or to be subsequently procured, the insured shall give notice to the insurer. > Ng Hua declared that there was non. The very next day, the building and the goods stored therein burned. > Subsequently, the claim of Ng Hua for the proceeds was denied by General since it discovered that Ng Hua had obtained an insurance from General Indemnity for the same goods and for the same period of time. Issue: Whether or not General Insurance can refuse to pay the proceeds. Held: Yes. Violation of the statement which is to be considered a warranty entitles the insurer to rescind the contract of insurance. Such misrepresentation is fatal. Vda. De Canilang v. CA - Concealment 223 SCRA 443 (1993) Facts: > Canilang consulted Dr. Claudio and was diagnosed as suffering from "sinus tachycardia." Mr. Canilang consulted the same doctor again on 3 August 1982 and this time was found to have "acute bronchitis."

> On the next day, 4 August 1982, Canilang applied for a "non-medical" insurance policy with Grepalife naming his wife, as his beneficiary. Canilang was issued ordinary life insurance with the face value of P19,700. > On 5 August 1983, Canilang died of "congestive heart failure," "anemia," and "chronic anemia." The wife as beneficiary, filed a claim with Grepalife which the insurer denied on the ground that the insured had concealed material information from it. > Vda Canilang filed a complaint with the Insurance Commissioner against Grepalife contending that as far as she knows her husband was not suffering from any disorder and that he died of kidney disorder. > Grepalife was ordered to pay the widow by the Insurance Commissioner holding that there was no intentional concealment on the Part of Canilang and that Grepalife had waived its right to inquire into the health condition of the applicant by the issuance of the policy despite the lack of answers to "some of the pertinent questions" in the insurance application. CA reversed. Issue: Whether or not Grepalife is liable. Held: SC took note of the fact that Canilang failed to disclose that hat he had twice consulted Dr. Wilfredo B. Claudio who had found him to be suffering from "sinus tachycardia" and "acute bronchitis. Under the relevant provisions of the Insurance Code, the information concealed must be information which the concealing party knew and "ought to [have] communicate[d]," that is to say, information which was "material to the contract. The information which Canilang failed to disclose was material to the ability of Grepalife to estimate the probable risk he presented as a subject of life insurance. Had Canilang disclosed his visits to his doctor, the diagnosis made and the medicines prescribed by such doctor, in the insurance application, it may be reasonably assumed that Grepalife would have made further inquiries and would have probably refused to issue a non-medical insurance policy or, at the very least, required a higher premium for the same coverage. The materiality of the information withheld by Canilang from Grepalife did not depend upon the state of mind of Jaime Canilang. A man's state of mind or subjective belief is not capable of proof in our judicial process, except through proof of external acts or failure to act from which inferences as to his subjective belief may be reasonably drawn. Neither does materiality depend upon the actual or physical events which ensue. Materiality relates rather to the "probable and reasonable influence of the facts" upon the party to whom the communication should have been made, in assessing the risk involved in making or omitting to make further inquiries and in accepting the application for insurance; that "probable and reasonable influence of the facts" concealed must, of course, be determined objectively, by the judge ultimately. SC found it difficult to take seriously the argument that Grepalife had waived inquiry into the concealment by issuing the insurance policy notwithstanding Canilang's failure to set out answers to some of the questions in the insurance application. Such failure precisely constituted concealment on the part of Canilang. Petitioner's argument, if accepted, would obviously erase Section 27 from the Insurance Code of 1978. Fieldmans Insurance v. Songco - Disclosure of Material Facts in Insurance 25 SCRA 70 Facts: > In 1960, Sambat, an agent of Fieldmans Insurance, induced Songco, a man of scant ed ucation to enter into a common carrier insurance contract with Fieldman. > During the inducement, a son of Songco butted in and said that they could not accept the type of insurance offered because theirs was an owner-type jeepney and not a common carrier. > Sambat answered that it did not matter because the insurance company was not owned by the government and therefore had nothing to do with rules and regulations of the latter (Fieldman). > The insurance was executed and approved for a year from Sept. 1960-1961. It was renewed in 1961 for another year. > In Oct. 1961, the jeepney collided with a car in Bulacan and as a result, Sonco died. The remaining members of the family claimed the proceeds of the insurance with the company but it refused to pay on the ground that the vehicle was not a common carrier. Issue:

Whether or not the Songcos can claim the insurance proceeds despite the fact that the vehicle concerned was an owner and not a common carrier. Held: Yes. The company is estopped from asserting that the vehicle was not covered. After it had led Federico Songco to believe that he could qualify under the common carrier liability insurance policy, and to enter into a contract of insurance paying the premiums due, it could not thereafter be permitted to change its stand to the detriment of the heirs of the insured. It knew all along that Frederico owned a private vehicle. Its agent Sambat twice exerted the utmost pressure on the insured, a man of scant education, and the company did not object to this. Insular Life v. Feliciano - Concealment 73 PHIL 201 Facts: > Evaristo Feliciano filed an application with Insular Life upon the solicitation of one of its agents. > It appears that during that time, Evaristo was already suffering from tuberculosis. Such fact appeared during the medical exam, but the examiner and the companys agent ignored it. > After that, Evaristo was made to sign an application form and thereafter the blank spaces were filled by the medical examiner and the agent making it appear that Evaristo was a fit subject of insurance. (Evaristo could not read and understand English) > When Evaristo died, Insular life refused to pay the proceeds because of concealment. Issue: Whether or not Insular Life was bound by their agents acts. Held: Yes. The insurance business has grown so vast and lucrative within the past century. Nowadays, even people of modest means enter into insurance contracts. Agents who solicit contracts are paid large commissions on the policies secured by them. They act as general representatives of insurance companies. IN the case at bar, the true state of health of the insured was concealed by the agents of the insurer. The insurers medical examiner approved the application knowing fully well that the applicant was sick. The situation is one in which of two innocent parties must bear a loss for his reliance upon a third person. In this case, it is the one who drafted and accepted the policy and consummated the contract. It seems reasonable that as between the two of them, the one who employed and gave character to the third person as its agent should be the one to bear the loss. Hence, Insular is liable to the beneficiaries. Insular life v. Feliciano 74 PHIL 4681 Facts: Insular life filed a motion for reconsideration of the decision in the preceding case. Issue: WON Insular Life was bound by their agents acts. Held: NO There was collusion between Evaristo and the agent and the medical examiner in making it appear that Evaristo was a fit subject for insurance. When Evaristo authorized them to write the answers for him, he made them his own agents for that purpose and he was responsible for their acts in that connection. If they falsified the answers for him, he could not evade liability for the falsification. He was not supposed to sign the application in blank. He knew that his answers would be the basis for the policy, and was required with his signature to vouch for their truth. The judgment rendered therefore in the preceding case is thus reversed, and Insular Life is absolved from liability.

Sun Life v. CA - Concealment in Insurance

245 SCRA 268 (1995) Facts: > On April 15, 1986, Bacani procured a life insurance contract for himself from Sun Life. He was issued a life insurance policy with double indemnity in case of accidental death. The designated beneficiary was his mother, Bernarda. > On June 26, 1987, the insured died in a plane crash. Bernarda Bacani filed a claim with Sun Life, seeking the benefits of the insurance. Sun Life conducted an investigation and its findings prompted it to reject the claim. > Sun Life discovered that 2 weeks prior to his application, Bacani was examined and confined at the Lung Center of the Philippines, where he was diagnosed for renal failure. During his confinement, the deceased was subjected to urinalysis, ultra-sonography and hematology tests. He did not reveal such fact in his application. > In its letter, Sun Life informed Berarda, that the insured did not disclosed material facts relevant to the issuance of the policy, thus rendering the contract of insurance voidable. A check representing the total premiums paid in the amount of P10,172.00 was attached to said letter. > Bernarda and her husband, filed an action for specific performance against Sun Life. RTC ruled for Bernarda holding that the facts concealed by the insured were made in good faith and under the belief that they need not be disclosed. Moreover, it held that the health history of the insured was immaterial since the insurance policy was "non-medical." CA affirmed. Issue: Whether or not the beneficiary can claim despite the concealment. Held: NOPE. Section 26 of the Insurance Code is explicit in requiring a party to a contract of insurance to communicate to the other, in good faith, all facts within his knowledge which are material to the contract and as to which he makes no warranty, and which the other has no means of ascertaining. Materiality is to be determined not by the event, but solely by the probable and reasonable influence of the facts upon the party to whom communication is due, in forming his estimate of the disadvantages of the proposed contract or in making his inquiries (The Insurance Code, Sec 31) The terms of the contract are clear. The insured is specifically required to disclose to the insurer matters relating to his health. The information which the insured failed to disclose were material and relevant to the approval and the issuance of the insurance policy. The matters concealed would have definitely affected petitioner's action on his application, either by approving it with the corresponding adjustment for a higher premium or rejecting the same. Moreover, a disclosure may have warranted a medical examination of the insured by petitioner in order for it to reasonably assess the risk involved in accepting the application. Thus, "good faith" is no defense in concealment. The insured's failure to disclose the fact that he was hospitalized for two weeks prior to filing his application for insurance, raises grave doubts about his bonafides. It appears that such concealment was deliberate on his part. Ng Gan Zee v. Asian Crusader Life - Imperfection in the Application Form 122 SCRA 61 Facts: > In 1962, Kwon Nam applied for a 20yr endowment insurance on his life with his wife, Ng Gan Zee as the beneficiary. > He stated in his application that he was operated on for tumor of the stomach associated with ulcer. > In 1963, Kwong died of cancer of the liver with metastasis. Asian refused to pay on the ground of alse information. > It was found that prior to his application, Kwong was diagnosed to have peptic ulcers, and that during the operation what was removed from Kwongs body was actually a portion of the stomach and not tumor. Issue: Whether or not the contract may be rescinded on the ground of the imperfection in the application form. Held: NO.

Kwong did not have sufficient knowledge as to distinguish between a tumor and a peptic ulcer. His statement therefore was made in good faith. Asian should have made an inquiry as to the illness and operation of Kwong when it appeared on the face of the application that a question appeared to be imperfectly answered. Asians failure to inquire constituted a waiver of the imperfection in the answer. Harding v. Commercial Union Assurance Company- Willful Misstatement 38 PHIL 464 Facts: > Henry Harding bought a car for 2T in 1915. He then gave the car to his wife Mrs. Harding. > While Mrs. Harding was having the car repaired at the Luneta Garage (Luneta was an agent of Smith Bell and Co., which in turn is Commercial Unions agent), the latter induced Mrs. Harding to insure the care with Commercial. > Mrs. Harding agreed, and Smith Bell sent an agent to Luneta Garage, who together with the manager of LUneta, appraised the car and declared that its present value was P3T. This amt was written in the proposal form which Mrs. Harding signed. > Subsequently, the car was damaged by fire. Commercial refused to pay because the cars present value was only 2.8T and not 3T. Issue: Whether or not Commercial is liable. Held: Commercial is liable. Where it appears that the proposal form, while signed by the insured was made out by the person authorized to solicit the insurance (Luneta and Smith Bell) the facts stated in the proposal, even if incorrect, will not be regarded as warranted by the insured, in the absence of willful misstatement. Under such circumstances, the proposal is to be regarded as the act of the insurer. Saturnino v. Philamlife - False Representation 7 SCRA 316 Facts: > 2 months prior to the insurance of the policy, Saturnino was operated on for cancer, involving complete removal of the right breast, including the pectoral muscles and the glands, found in the right armpit. > Notwithstanding the fact of her operation, Saturnino did not make a disclosure thereof in her application for insurance. > She stated therein that she did not have, nor had she ever had, among others listed in the application, cancer or other tumors; that she had not consulted any physician, undergone any operation or suffered any injury within the preceding 5 years. > She also stated that she had never been treated for, nor did she ever have any illness or disease peculiar to her sex, particularly of the breast, ovaries, uterus and menstrual disorders. > The application also recited that the declarations of Saturnino constituted a further basis for the issuance of the policy. Issue: Whether or not the insured made such false representation of material facts as to avoid the policy. Held: YES. There can be no dispute that the information given by her in the application for insurance was false, namely, that she never had cancer or tumors or consulted any physician or undergone any operation within the preceding period of 5 years. The question to determine is: Are the facts then falsely represented material? The Insurance Law provides that materiality is to be determined not by the event, but solely by the probable and reasonable influence of the facts upon the party to whom the communication is due, in forming his estimate of the proposed contract, or making his inquiries.

The contention of appellants is that the facts subject of the representation were not material in view of the nonmedical nature of the insurance applied for, which does away with the usual requirement of medical examination before the policy is issued. The contention is without merit. If anything, the waiver of medical examination renders even more material the information required of the applicant concerning previous condition of health and diseases suffered, for such information necessarily constitutes an important factor which the insurer takes into consideration in deciding whether to issue the policy or not. Appellants also contend that there was no fraudulent concealment of the truth inasmuch as the insured herself did not know, since her doctor never told her, that the disease for which she had been operated on was cancer. In the first place, concealment of the fact of the operation itself was fraudulent, as there could not have been any mistake about it, no matter what the ailment. Secondly, in order to avoid a policy, it is not necessary to show actual fraud on the part of the insured. In this jurisdiction, concealment, whether intentional or unintentional entitled the insurer to rescind the contract of insurance, concealment being defined as negligence to communicate that which a party knows and ought to communicate. The basis of the rule vitiating the contract in cases of concealment is that it misleads or deceives the insurer into accepting the risk, or accepting it at a rate of premium agreed upon. The insurer, relying upon the belief that the insured will disclose every material fact within his actual or presumed knowledge, is misled into a belief that the circumstances withheld does not exist, and he is thereby induced to estimate the risk upon a false basis that it does not exist. Edillon v. Manila Bankers Life Insurance Corp. - Concealment 117 SCRA 187 Facts: > In Apr. 1969, Carmen Lapuz applied for insurance with Manila Bankers. In the application she stated the date of her birth as July 11, 1904 (around 64 yrs old). The policy was thereafter issued. > Subsequently, in May 1969, Carmen died of a car accident. Her sister, as beneficiary claimed the proceeds of the insurance. > Manila Bankers refused to pay because the certificate of insurance contained a provision excluding its liability to pay claims to persons under 16 or over 60. Issue: Whether or not the policy is void considering that the insured was over 60 when she applied. Held: NO. The age of Carmen was not concealed to the insurance company. Her application form indicated her true age. Despite such information, Manila Bankers accepted the premium and issued the policy. It had all the time to process the application and notice the applicants age. If it failed to act, it was because Manila Bankers was willing to waive such disqualifications or it simply overlooked such fact. It is therefore estopped from disclaiming any liability. Philamcare v. CA- Health Care Agreement 379 SCRA 356 (2002) Facts: > Ernani Trinos, applied for a health care coverage with Philamcare. In the standard application form, he answered NO to the following question: Have you or any of your family members ever consulted or been treated for high blood pressure, heart trouble, diabetes, cancer, liver disease, asthma or peptic ulcer? (If Yes, give details) > The application was approved for a period of one year from March 1, 1988 to March 1, 1989. He was a issued Health Care Agreement, and under such, he was entitled to avail of hospitalization benefits, whether ordinary or emergency, listed therein. He was also entitled to avail of "out-patient benefits" such as annual physical examinations, preventive health care and other out-patient services. > Upon the termination of the agreement, the same was extended for another year from March 1, 1989 to March 1, 1990, then from March 1, 1990 to June 1, 1990. The amount of coverage was increased to a maximum sum of P75,000.00 per disability.

> During the period of his coverage, Ernani suffered a heart attack and was confined at the Manila Medical Center (MMC) for one month beginning March 9, 1990. > While her husband was in the hospital, Julita tried to claim the benefits under the health care agreement. However, Philamcare denied her claim saying that the Health Care Agreement was void. > According to Philamcare, there was concealment regarding Ernani's medical history. Doctors at the MMC allegedly discovered at the time of Ernani's confinement that he was hypertensive, diabetic and asthmatic, contrary to his answer in the application form. > Julita had no choice but to pay the hospitalization expenses herself, amounting to about P76,000.00 > After her husband was discharged from the MMC, he was attended by a physical therapist at home. Later, he was admitted at the Chinese General Hospital (CGH). Due to financial difficulties, Julita brought her husband home again. In the morning of April 13, 1990, Ernani had fever and was feeling very weak. Julita was constrained to bring him back to the CGH where he died on the same day. > Julita instituted, an action for damages against Philamcare. She asked for reimbursement of her expenses plus moral damages and attorney's fees. RTC decided in favor of Julita. CA affirmed. Issues and Resolutions: Philamcare brought the instant petition for review, raising the primary argument that a health care agreement is not an insurance contract; hence the "incontestability clause" under the Insurance Code Title 6, Sec. 48 does not apply. SC held that in the case at bar, the insurable interest of respondent's husband in obtaining the health care agreement was his own health. The health care agreement was in the nature of non-life insurance, which is primarily a contract of indemnity. Once the member incurs hospital, medical or any other expense arising from sickness, injury or other stipulated contingent, the health care provider must pay for the same to the extent agreed upon under the contract. Under the title Claim procedures of expenses, Philamcare. had 12 mos from the date of issuance of the Agreement within which to contest the membership of the patient if he had previous ailment of asthma, and six months from the issuance of the agreement if the patient was sick of diabetes or hypertension. The periods having expired, the defense of concealment or misrepresentation no longer lie. Petitioner argues that respondent's husband concealed a material fact in his application. It appears that in the application for health coverage, petitioners required respondent's husband to sign an express authorization for any person, organization or entity that has any record or knowledge of his health to furnish any and all information relative to any hospitalization, consultation, treatment or any other medical advice or examination. Philamcare cannot rely on the stipulation regarding "Invalidation of agreement" which reads: Failure to disclose or misrepresentation of any material information by the member in the application or medical examination, whether intentional or unintentional, shall automatically invalidate the Agreement from the very beginning and liability of Philamcare shall be limited to return of all Membership Fees paid. An undisclosed or misrepresented information is deemed material if its revelation would have resulted in the declination of the applicant by Philamcare or the assessment of a higher Membership Fee for the benefit or benefits applied for. The answer assailed by petitioner was in response to the question relating to the medical history of the applicant. This largely depends on opinion rather than fact, especially coming from respondent's husband who was not a medical doctor. Where matters of opinion or judgment are called for, answers made in good faith and without intent to deceive will not avoid a policy even though they are untrue. Thus, (A)lthough false, a representation of the expectation, intention, belief, opinion, or judgment of the insured will not avoid the policy if there is no actual fraud in inducing the acceptance of the risk, or its acceptance at a lower rate of premium, and this is likewise the rule although the statement is material to the risk, if the statement is obviously of the foregoing character, since in such case the insurer is not justified in relying upon such statement, but is obligated to make further inquiry. There is a clear distinction between such a case and one in which the insured is fraudulently and intentionally states to be true, as a matter of expectation or belief, that which he then knows, to be actually untrue, or the impossibility of which is shown by the facts within his knowledge, since in such case the intent to deceive the insurer is obvious and amounts to actual fraud. The fraudulent intent on the part of the insured must be established to warrant rescission of the insurance contract. Concealment as a defense for the health care provider or insurer to avoid liability is an affirmative defense and the duty to establish such defense by satisfactory and convincing evidence rests upon the provider or insurer. In any case, with or without the authority to investigate, petitioner is liable for claims made under the contract. Having

assumed a responsibility under the agreement, petitioner is bound to answer the same to the extent agreed upon. In the end, the liability of the health care provider attaches once the member is hospitalized for the disease or injury covered by the agreement or whenever he avails of the covered benefits which he has prepaid. Under Section 27 of the Insurance Code, "a concealment entitles the injured party to rescind a contract of insurance." The right to rescind should be exercised previous to the commencement of an action on the contract. In this case, no rescission was made. Besides, the cancellation of health care agreements as in insurance policies require the concurrence of the following conditions: 1. Prior notice of cancellation to insured; 2. Notice must be based on the occurrence after effective date of the policy of one or more of the grounds mentioned; 3. Must be in writing, mailed or delivered to the insured at the address shown in the policy; 4. Must state the grounds relied upon provided in Section 64 of the Insurance Code and upon request of insured, to furnish facts on which cancellation is based. None of the above pre-conditions was fulfilled in this case. When the terms of insurance contract contain limitations on liability, courts should construe them in such a way as to preclude the insurer from non-compliance with his obligation. Being a contract of adhesion, the terms of an insurance contract are to be construed strictly against the party which prepared the contract the insurer. By reason of the exclusive control of the insurance company over the terms and phraseology of the insurance contract, ambiguity must be strictly interpreted against the insurer and liberally in favor of the insured, especially to avoid forfeiture. This is equally applicable to Health Care Agreements. The phraseology used in medical or hospital service contracts, such as the one at bar, must be liberally construed in favor of the subscriber, and if doubtful or reasonably susceptible of two interpretations the construction conferring coverage is to be adopted, and exclusionary clauses of doubtful import should be strictly construed against the provider. Soliman v. US Life- Rescind Contract of Insurance 104 PHIL 1046 Facts: > US Life issued a 20 yr endowment life policy on the joint lives of Patricio Soliman and his wife Rosario, each of them being the beneficiary of the other. > In Mar. 1949, the spouses were informed that the premium for Jan 1949 was still unpaid notwithstanding that the 31-day grace period has already expired, and they were furnished at the same time long-form health certificates for the reinstatement of the policies. > In Apr 1949, they submitted the certificates and paid the premiums. > In Jan. 1950, Rosario died of acute dilation of the heart, and thereafter, Patricio filed a claim for the proceeds of the insurance. > US life denied the claim and filed for the rescission of the contract on the ground that the certificates failed to disclose that Rosario had been suffering from bronchial asthma for 3 years prior to their submission. Issue: Whether or not the contract can still be rescinded. Held: Yes. The insurer is once again given two years from the date of reinstatement to investigate into the veracity of the facts represented by the insured in the application for reinstatement. When US life sought to rescind the contract on the ground of concealment/misrepresentation, two years had not yet elapsed. Hence, the contract can still be rescinded. Enriquez v. SunLife- Insurance Policy 41 PHIL 269 Facts: > On Sept. 24 1917, Herrer made an application to SunLife through its office in Manila for life annuity. > 2 days later, he paid the sum of 6T to the companys anager in its Manila office an d was given a receipt. > On Nov. 26, 1917, the head office gave notice of acceptance by cable to Manila. On the same date, the Manila office prepared a letter notifying Herrer that his application has been accepted and this was placed in the ordinary channels of transmission, but as far as known was never actually mailed and never received by Herrer.

> Herrer died on Dec. 20, 1917. The plaintiff as administrator of Herrers estate brought this action to recover the 6T paid by the deceased. Issue: Whether or not the insurance contract was perfected. Held: NO. The contract for life annuity was NOT perfected because it had NOT been proved satisfactorily that the acceptance of the application ever came to the knowledge of the applicant. An acceptance of an offer of insurance NOT actually or constructively communicated to the proposer does NOT make a contract of insurane, as the locus poenitentiae is ended when an acceptance has passed beyond the control of the party. NOTE: Life annuity is the opposite of a life insurance. In life annuity, a big amount is given to the insurance company, and if after a certain period of time the insured is stil living, he is entitled to regular smaller amounts for the rest of his life. Examples of Life annuity are pensions. Life Insurance on the other hand, the insured during the period of the coverage makes small regular payments and upon his death, the insurer pays a big amount to his beneficiaries. Perez v. CA- Perfection of the Contract of Insurance 323 SCRA 613 (2000) Facts: > Primitivo Perez had been insured with the BF Lifeman Insurance Corporation since 1980 for P20,000.00. > In October 1987, an agent of Lifeman, Rodolfo Lalog, visited Perez in Quezon and convinced him to apply for additional insurance coverage of P50,000.00, to avail of the ongoing promotional discount of P400.00 if the premium were paid annually. > Primitivo B. Perez accomplished an application form for the additional insurance coverage. Virginia A. Perez, his wife, paid P2,075.00 to Lalog. The receipt issued by Lalog indicated the amount received was a "deposit." > Unfortunately, Lalog lost the application form accomplished by Perez and so on October 28, 1987, he asked the latter to fill up another application form. On November 1, 1987, Perez was made to undergo the required medical examination, which he passed. > Lalog forwarded the application for additional insurance of Perez, together with all its supporting papers, to the office of BF Lifeman Insurance Corporationn in Quezon which office was supposed to forward the papers to the Manila office. > On November 25, 1987, Perez died while he was riding a banca which capsized during a storm. > At the time of his death, his application papers for the additional insurance were still with the Quezon office. Lalog testified that when he went to follow up the papers, he found them still in the Quezon office and so he personally brought the papers to the Manila office of BF Lifeman Insurance Corporation. It was only on November 27, 1987 that said papers were received in Manila. > Without knowing that Perez died on November 25, 1987, BF Lifeman Insurance Corporation approved the application and issued the corresponding policy for the P50,000.00 on December 2, 1987 > Virginia went to Manila to claim the benefits under the insurance policies of the deceased. She was paid P40,000.00 under the first insurance policy for P20,000.00 (double indemnity in case of accident) but the insurance company refused to pay the claim under the additional policy coverage of P50,000.00, the proceeds of which amount to P150,000.00 in view of a triple indemnity rider on the insurance policy. > In its letter of January 29, 1988 to Virginia A. Perez, the insurance company maintained that the insurance for P50,000.00 had not been perfected at the time of the death of Primitivo Perez. Consequently, the insurance company refunded the amount of P2,075.00 which Virginia Perez had paid > Lifeman filed for the rescission and the declaration of nullity. Perez, on the other hand, averred that the deceased had fulfilled all his prestations under the contract and all the elements of a valid contract are present. > RTC ruled in favor of Perez. CA reversed. Issue: Whether or not there was a perfected additional insurance contract. Held:

The contract was not perfected. Insurance is a contract whereby, for a stipulated consideration, one party undertakes to compensate the other for loss on a specified subject by specified perils. A contract, on the other hand, is a meeting of the minds between two persons whereby one binds himself, with respect to the other to give something or to render some service. Consent must be manifested by the meeting of the offer and the acceptance upon the thing and the cause which are to constitute the contract. The offer must be certain and the acceptance absolute. When Primitivo filed an application for insurance, paid P2,075.00 and submitted the results of his medical examination, his application was subject to the acceptance of private respondent BF Lifeman Insurance Corporation. The perfection of the contract of insurance between the deceased and respondent corporation was further conditioned upon compliance with the following requisites stated in the application form: "there shall be no contract of insurance unless and until a policy is issued on this application and that the said policy shall not take effect until the premium has been paid and the policy delivered to and accepted by me/us in person while I/We, am/are in good health." The assent of private respondent BF Lifeman Insurance Corporation therefore was not given when it merely received the application form and all the requisite supporting papers of the applicant. Its assent was given when it issues a corresponding policy to the applicant. Under the abovementioned provision, it is only when the applicant pays the premium and receives and accepts the policy while he is in good health that the contract of insurance is deemed to have been perfected. It is not disputed, however, that when Primitivo died on November 25, 1987, his application papers for additional insurance coverage were still with the branch office of respondent corporation in Gumaca and it was only two days later, or on November 27, 1987, when Lalog personally delivered the application papers to the head office in Manila. Consequently, there was absolutely no way the acceptance of the application could have been communicated to the applicant for the latter to accept inasmuch as the applicant at the time was already dead. CIR v. Lincoln Phil Life - Automatic Increase Clause 379 SCRA 423 (2002) Facts: > In the years prior to 1984, Lincoln issued a special kind of life insurance policy known as the "Junior Estate Builder Policy," the distinguishing feature of which is a clause providing for an automatic increase in the amount of life insurance coverage upon attainment of a certain age by the insured without the need of issuing a new policy. The clause was to take effect in the year 1984. > Documentary stamp taxes due on the policy were paid to the petitioner only on the initial sum assured. > Subsequently, petitioner issued deficiency documentary stamps tax assessment for the year 1984, corresponding to the amount of automatic increase of the sum assured on the policy issued by respondent. > Lincoln questioned the deficiency assessments and sought their cancellation in a petition filed in the Court of Tax Appeals. CTA found no basis for the assessment. CA affirmed. Issue: Whether or not the automatic increase of the sum assured on the policy is taxable. Held: YES. CIR claims that the "automatic increase clause" in the subject insurance policy is separate and distinct from the main agreement and involves another transaction; and that, while no new policy was issued, the original policy was essentially re-issued when the additional obligation was assumed upon the effectivity of this "automatic increase clause" in 1984; hence, a deficiency assessment based on the additional insurance not covered in the main policy is in order. The SC agreed with this contention. The subject insurance policy at the time it was issued contained an "automatic increase clause." Although the clause was to take effect only in 1984, it was written into the policy at the time of its issuance. The distinctive feature of the "junior estate builder policy" called the "automatic increase clause" already formed part and parcel of the insurance contract, hence, there was no need for an execution of a separate agreement for the increase in the coverage that took effect in 1984 when the assured reached a certain age.

It is clear from Section 173 of the NIRC that the payment of documentary stamp taxes is done at the time the act is done or transaction had and the tax base for the computation of documentary stamp taxes on life insurance policies under Section 183 of NIRC is the amount fixed in policy, unless the interest of a person insured is susceptible of exact pecuniary measurement. Logically, we believe that the amount fixed in the policy is the figure written on its face and whatever increases will take effect in the future by reason of the "automatic increase clause" embodied in the policy without the need of another contract. Here, although the automatic increase in the amount of life insurance coverage was to take effect later on, the date of its effectivity, as well as the amount of the increase, was already definite at the time of the issuance of the policy. Thus, the amount insured by the policy at the time of its issuance necessarily included the additional sum covered by the automatic increase clause because it was already determinable at the time the transaction was entered into and formed part of the policy. The "automatic increase clause" in the policy is in the nature of a conditional obligation under Article 1181, 8 by which the increase of the insurance coverage shall depend upon the happening of the event which constitutes the obligation. In the instant case, the additional insurance that took effect in 1984 was an obligation subject to a suspensive obligation, 9 but still a part of the insurance sold to which private respondent was liable for the payment of the documentary stamp tax. Lim v. Sun Life 41 PHIL 263 Facts: > On July 6, 1917, Luis Lim Y Garcia of Zamboanga applied for a policy of life insurance with Sunlife in the amount of 5T. > He designated his wife Pilar Lim as the beneficiary. The first premium of P433 was paid by Lim and company issued a provisional policy > Such policy contained the following provisions xx the abovementioned life is to be assured in accordance with the terms and conditions contained or inserted by the Company in the policy which may be granted by it in this particular case for 4 months only from the date of the application, PROVIDED that the company shall confirm this agreement by issuing a policy on said application xxx. Should the company NOT issue such a policy, then this agreement shall be null and void ab initio and the Company shall be held not to have been on the risk at all, but in such case, the amount herein shall be returned. > Lim died on Aug. 23, 1917 after the issuance of the provisional policy but before the approval of the application by the home office of the insurance company. > The instant action is brought by the beneficiary to recover from Sun Life the sum of 5T. Issue: Whether or not the beneficiary can collect the 5T. Held: NO. The contract of insurance was not consummated by the parties. The above quoted agreement clearly stated that the agreement should NOT go into effect until the home office of the Company shall confirm it by issuing a policy. It was nothing but an acknowledgment by the Company that it has received a sum of money agreed upon as the first years premium upon a policy to be issued upon the a pplication if it is accepted by the Company. When an agreement is made between the applicant and the agent whether by signing an application containing such condition or otherwise, that no liability shall attach until the principal approves the risk and a receipt is given by the agent, such acceptance is merely conditional and is subordinated to the companys act in approving or rejecting; so in life insurance a binding slip or receipt does not insure itself. Bonifacio Bros. v. Mora 20 SCRA 262 Facts: > Enrique Mora mortgaged his Odlsmobile sedan car to HS Reyes Inc. with the condition that Mora would insure the car with HS Reyes as beneficiary.

> The car was then insured with State Insurance Company and the policy delivered to Mora. > During the effectivity of the insurance contract, the car figured in an accident. The company then assigned the accident to an insurance appraiser for investigation and appraisal of the damage. > Mora without the knowledge and consent of HS Reyes, authorized Bonifacio Bros to fix the car, using materials supplied by the Ayala Auto Parts Company. > For the cost of Labor and materials, Mora was billed P2,102.73. The bill was sent to the insurers appraiser. The insurance company drew a check in the amount of the insurance proceeds and entrusted the check to its appraiser for delivery to the proper party. > The car was delivered to Mora without the consent of HS Reyes, and without payment to Bonifacio Bros and Ayala. > Upon the theory that the insurance proceeds should be directly paid to them, Bonifacio and Ayala filed a complaint against Mora and the insurer with the municipal court for the collection of P2,102.73. > The insurance company filed its answer with a counterclaim for interpleader, requiring Bonifacio and HS Reyes to interplead in order to determine who has a better right to the proceeds. Issue: Whether or not there is privity of contract between Bonficacio and Ayala on one hand and State Insurance on the other. Held: NONE. It is fundamental that contracts take effect only between the parties thereto, except in some specific instance provided by law where the contract contains some stipulation in favor of a third person. Such stipulation is known as a stipulation pour autrui; or a provision in favor of a third person not a party to the contract. Under this doctrine, a third person is ed to avail himself of a benefit granted to him by the terms of the contract, provided that the contracting parties have clearly and deliberately conferred a favor upon such person. Consequently, a third person NOT a party to the contract has NO action against the aprties thereto, and cannot generally demand the enforcement of the same. The question of whether a third person has an enforceable interest in a contract must be settled by determining whether the contracting parties intended to tender him such an interest by deliberately inserting terms in their agreement with the avowed purpose of conferring favor upon such third person. IN this connection, this court has laid down the rule that the fairest test to determine whether the interest of a 3 rd person in a contract is a stipulation pour autrui or merely an incidental interest, is to rely upon the intention of the parties as disclosed by their contract. In the instant case the insurance contract does not contain any words or clauses to disclose an intent to give any benefit to any repairmen or material men in case of repair of the car in question. The parties to the insurance contract omitted such stipulation, which is a circumstance that supports the said conclusion. On the other hand, the "loss payable" clause of the insurance policy stipulates that "Loss, if any, is payable to H.S. Reyes, Inc." indicating that it was only the H.S. Reyes, Inc. which they intended to benefit. A policy of insurance is a distinct and independent contract between the insured and insurer, and third persons have no right either in a court of equity, or in a court of law, to the proceeds of it, unless there be some contract of trust, expressed or implied, by the insured and third person. In this case, no contract of trust, express or implied. In this case, no contract of trust, expressed or implied exists. We, therefore, agree with the trial court that no cause of action exists in favor of the appellants in so far as the proceeds of insurance are concerned. The appellant's claim, if at all, is merely equitable in nature and must be made effective through Enrique Mora who entered into a contract with the Bonifacio Bros Inc. This conclusion is deducible not only from the principle governing the operation and effect of insurance contracts in general, but is clearly covered by the express provisions of section 50 of the Insurance Act (now Sec. 53). The policy in question has been so framed that "Loss, if any, is payable to H. S. Reyes, Inc." which unmistakably shows the intention of the parties. Coquia v. Fieldmens Insurance 26 SCRA 172

Facts: > On Dec. 1, 1961, Fieldmens Insurance co. Issued in favor of the Manila Yellow Taxicab a common carrier insurance policy with a stipulation that the company shall indemnify the insured of the sums which the latter wmy be held liable for with respect to death or bodily injury to any faire-paying passenger including the driver and conductor. > The policy also stated that in the event of the death of the driver, the Company shall indemnify his personal representatives and at the Companys option may make indemnity payable directly to the claimants or heirs of th e claimants. > During the policys lifetime, a taxicab of the insured driven by Coquia met an accident and Coquia died. > When the company refused to pay the only heirs of Coquia, his parents, they institued this complaint. The company contends that plaintiffs have no cause of action since the Coquias have no contractual relationship with the company. Issue: Whether or not plaintiffs have the right to collect on the policy. Held: YES. Athough, in general, only parties to a contract may bring an action based thereon, this rule is subject to exceptions, one of which is found in the second paragraph of Article 1311 of the Civil Code of the Philippines, reading: " If a contract should contain some stipulation in favor of a third person, he may demand its fulfillment provided he communicated his acceptance to the obligor before its revocation. A mere incidental benefit or interest of a person is not sufficient. The contracting parties must have clearly and deliberately conferred a favor upon a third person." This is but the restatement of a well-known principle concerning contracts pour autrui, the enforcement of which may be demanded by a third party for whose benefit it was made, although not a party to the contract, before the stipulation in his favor has been revoked by the contracting parties In the case at bar, the policy under consideration is typical of contracts pour autrui this character being made more manifest by the fact that the deceased driver paid fifty percent (50%) of the corresponding premiums, which were deducted from his weekly commissions. Under these conditions, it is clear that the Coquias who, admittedly, are the sole heirs of the deceased have a direct cause of action against the Company, and, since they could have maintained this action by themselves, without the assistance of the insured it goes without saying that they could and did properly join the latter in filing the complaint herein. SSS v. Davac - SSS Benefits 17 SCRA 863 Facts: > Davac was an SSS member, and designated Candelaria Davac, his alleged wife, as his beneficiary. > When he died, both his first wife, Lourdes and his second wife, Candelaria filed claims for the death benefits. > Due to the conflicting claims, the SSS filed a petition praying that both of them be required to interplead and litigate the conflicting claims. > The death benefits were awarded to Candelaria Davac. Issue: Who is entitled to the SSS benefits? Held: Candelaria. Under the SSS Act, the beneficiary as recorded by the employees employer is the one entitled to the death benefits, hence they should go to Candelaria. Lourdes contends that the designation made in the person of Candelaria who is party in a bigamous marriage is null and void for being against Art. 739 of the CC. SC held that the disqualification mentioned in Art. 739 is NOT applicable to Candelaria, because she was not guilty of concubinage , there bieing NO proof that she had actual knowledge of the previous marriage of her husband. Vda. De Consuegra v. GSIS - Retirement Insurance Benefits

37 SCRA 315 Facts: > Jose Consuegra was employed as a shop foreman of the Office of the District Engineer in Surigao Del Norte. > When he was still alive, he contracted two marriages: o First Rosario Diaz; 2 children = Jose Consuegra Jr. and Pedro but both predeceased him o 2nd Basilia Berdin; 7 children. (this was contracted in GF while the first marriage subsisted) > Being a GSIS member when he died, the proceeds of his life insurance were paid by the GSIS to Berdin and her children who were the beneficiaries named in the policy. > Since he was in the govt service for 22.5028 years, he was entitled to retirement insurance benefits, for which no beneficiary was designated. > Both families filed their claims with the GSIS, which ruled that the legal heirs were Diaz who is entitled to onehalf or 8/16 of the retirement benefits and Berdin and her children were entitled to the remaining half, each to receive an equal share of 1/16. > Berdin went to CFI on appeal. CFI affirmed GSIS decision. Issue: To whom should the retirement insurance benefits be paid? Held: Both families are entitled to half of the retirement benefits. The beneficiary named in the life insurance does NOT automatically become the beneficiary in the retirement insurance. When Consuegra, during the early part of 1943, or before 1943, designated his beneficiaries in his life insurance, he could NOT have intended those beneficiaries of his life insurance as also the beneficiaries of his retirement insurance because the provisions on retirement insurance under the GSIS came about only when CA 186 was amended by RA 660 on June 18, 1951. Sec. 11(b) clearly indicates that there is need for the employee to file an application for retirement insurance benefits when he becomes a GSIS member and to state his beneficiary. The life insurance and the retirement insurance are two separate and distinct systems of benefits paid out from 2 separate and distinct funds. In case of failure to name a beneficiary in an insurance policy, the proceeds will accrue to the estate of the insured. And when there exists two marriages, each family will be entitled to one-half of the estate. Heirs of Loreto C. Maramag v Maramag (2009) FACTS: Loreto Maramag designated as beneficiary his concubine Eva de Guzman Maramag Vicenta Maramag and Odessa, Karl Brian, and Trisha Angelie (heirs of Loreto Maramag) and his concubine Eva de Guzman Maramag, also suspected in the killing of Loreto and his illegitimate children are claiming for his insurance. Vicenta alleges that Eva is disqualified from claiming RTC: Granted - civil code does NOT apply CA: dismissed the case for lack of jurisdiction for filing beyond reglementary period ISSUE: W/N Eva can claim even though prohibited under the civil code against donation HELD: YES. Petition is DENIED. Any person who is forbidden from receiving any donation under Article 739 cannot be named beneficiary of a life insurance policy of the person who cannot make any donation to him If a concubine is made the beneficiary, it is believed that the insurance contract will still remain valid, but the indemnity must go to the legal heirs and not to the concubine, for evidently, what is prohibited under Art. 2012 is the naming of the improper beneficiary. SECTION 53. The insurance proceeds shall be applied exclusively to the proper interest of the person in whose name or for whose benefit it is made unless otherwise specified in the policy.

GR: only persons entitled to claim the insurance proceeds are either the insured, if still alive; or the beneficiary, if the insured is already deceased, upon the maturation of the policy. EX: situation where the insurance contract was intended to benefit third persons who are not parties to the same in the form of favorable stipulations or indemnity. In such a case, third parties may directly sue and claim from the insurer It is only in cases where the insured has not designated any beneficiary, or when the designated beneficiary is disqualified by law to receive the proceeds, that the insurance policy proceeds shall redound to the benefit of the estate of the insured Development Insurance v IAC G.R. No. 71360 July 16, 1986 Facts: A fire occurred in the building of Philippine Union. It sued for recovery of damages from the petitioner on the basis of an insurance contract between them. The petitioner failed to answer on time despite the numerous extensions it asked for. It was declared in default by the trial court. A judgment of default was subsequently rendered on the strength of the evidence given by the private respondent, which was allowed damages. The petitioner moved to lift the order of default. Its motion was denied. It went to the appellate court, which affirmed the decision of the trial court. Hence this appeal. Issue: Was Philippine Union required to jointly indemnify the building? Held: No. Petition dismissed. Ratio: The policy insured the private respondent's building against fire for P2,500,000.00. The petitioner argued that the respondent must share the difference between that amount and the face value of the policy and the loss sustained for 5.8 million under Condition 17 of the policy. The building was insured at P2,500,000.00 by agreement of the insurer and the insured. The agreement is known as an open policy and is subject to the express condition that: In the event of loss, whether total or partial, it is understood that the amount of the loss shall be subject to appraisal and the liability of the company, if established, shall be limited to the actual loss, subject to the applicable terms, conditions, warranties and clauses of this Policy, and in no case shall exceed the amount of the policy. Section 60 of the Insurance Code defines an open policy is one in which the value of the thing insured is not agreed upon but is left to be ascertained in case of loss." This means that the actual loss, as determined, will represent the total indemnity due the insured from the insurer except only that the total indemnity shall not exceed the face value of the policy. The actual loss has been ascertained in this case. Hence, applying the open policy clause as expressly agreed upon, the private respondent is entitled to indemnity in the total amount of P508,867.00. The refusal of its vice-president to receive the private respondent's complaint was the first indication of the petitioner's intention to prolong this case and postpone the discharge of its obligation to the private respondent under this agreement. They still evaded payment for 5 years. ACCFA v. Alpha Insurance | Reyes, J. July 29, 1968| NATURE Petition for review on certiorari FACTS - To guarantee the Asingan Farmers' Cooperative Marketing Association, Inc. (FACOMA) against loss on account of personal dishonesty, amounting to larceny/estafa of its Secretary-Treasurer, Ladines, appellee Alpha Insurance & Surety Company had issued, on 14 February 1958, its bond with Ladines as principal and the appellee as solidary surety. On the same date, the Asingan FACOMA assigned its rights to the appellant, Agricultural Credit Cooperative and Financing Administration (ACCFA) with approval of the principal and the surety. - During the effectivity of the bond, Ladines converted and misappropriated, to his personal benefit, some of the FACOMA funds, of which a part belonged to the ACCFA. Upon discovery of the loss, ACCFA immediately notified in writing the survey company on 10 October 1958, and presented the proof of loss within the period fixed in the bond; but despite repeated demands the surety company refused and failed to pay. ACCFA filed suit against

appellee on 30 May 1960. - Defendant Alpha Insurance & Surety Co., Inc., (now appellee) moved to dismiss the complaint as it was filed more than one year after plaintiff made claim for loss, contrary to the eighth condition of the bond - At first, the Court of First Instance denied dismissal; but, upon reconsideration, the court reversed its original stand, and dismissed the complaint on the ground that the action was filed beyond the contractual limitation period. Hence, this appeal. ISSUES & ARGUMENTS WON the provision of a fidelity bond that no action shall be had or maintained thereon unless commenced within one year from the making of a claim for the loss upon which the action is based, is valid, in view of Section 61-A of the Insurance Act invalidating stipulations limiting the time for commencing an action thereon to less than one year from the time the cause of action accrues? NO RATIONALE -A fidelity bond is, in the nature of a contract of insurance against loss from misconduct, and is governed by the same principles of interpretation. Consequently, the condition of the bond in question, limiting the period for bringing action is subject to the provisions of Section 61-A of the Insurance Act (No. 2427), as amended by Act 4101 of the pre-Commonwealth Philippine Legislature, prescribing that: SEC. 61-A: A condition, stipulation or agreement in any policy of insurance, limiting the time for commencing an action thereunder to a period of less than one year from the time when the cause of action accrues is void. - Since a "cause of action" requires, as essential elements, not only a legal right of the plaintiff and a correlative obligation of the defendant but also "an act or omission of the defendant in violation of said legal right," the cause of action does not accrue until the party obligated refuses, expressly or impliedly, to comply with its duty (in this case, to pay the amount of the bond). -The year for instituting action in court must be reckoned from the time of appellee's refusal to comply with its bond. It cant be counted from the creditor's filing of the claim of loss, for that does not import that the surety company will refuse to pay. -In so far, therefore, as condition eight of the bond requires action to be filed within one year from the filing of the claim for loss, such stipulation contradicts the public policy expressed in Section 61-A of the Philippine Insurance Act. - Condition eight of the bond, therefore, is null and void, and the appellant is not bound to comply with its provisions. The discouraging of unnecessary litigation must be deemed a rule of public policy, considering the unrelieved congestion in the courts. -As a consequence, the action may be brought within the statutory period of limitation for written contracts (New Civil Code, Article 1144). SPS. ANG (Paulo & Sally) v. Fulton Fire Insurance Co.PAULO ANG and SALLY C. ANG, plaintiffsappellees, FACTS:On September 9, 1953, Fulton Fire Insurance Company issued a Fire policy in favor of P. & S Department Store (Sally C. Ang) over stocks of general merchandise,consisting principally of dry goods, which were contained in a building occupiedby the Sps. at Laoag, Ilocos Norte. The premium is P500.00 annually. The insurance was issued for one year, but the same was renewed for another year on September 31, 1954. 3 months after, the store containing the goods insured was destroyed by fire. The Sps. then first claim form together with all the necessary papers relating, which was denied on April 6, 1956. It should also be noted that on January 13, 1955, Paulo Ang and 10 others were charged for arson but was acquitted.Spouses Ang instituted this action against the Fulton Fire Insurance Company andthe Paramount Surety and Insurance Company, Inc. to recover from them the facevalue of a fire insurance policy issued in plaintiffs' favor covering a store owned and operated by them in Laoag, Ilocos Norte. Fulton Fire Insurance Companyalleged that the loss by the fire was not accidental and was occasioned by the willful act of the plaintiff Paulo Ang himself. It claims that under paragraph 13of the policy, if the loss or damage is occasioned by the willful act of the insured, or if the claim is made and rejected but no action is commenced within 12months after such rejection, all benefits under the policy would be forfeited,and that since the claim of the plaintiffs was denied and plaintiffs received notice of denial on April 18, 1956, and they brought the action only on May 5, 1958, all the benefits under the policy have been forfeited. The Sps. alleged thatthey instituted a civil case (Paramount Surety & Insurance Co. - dropped as a defendant) to assert the claim on May 11, 1956, but was

dismissed without prejudice on September 3, 1957. If such period is to be deducted, the present action wasstill within the 12 month period.CFI: decision in favor of the plaintiffs. ISSUE:WON the filing of the previous suit tolled or suspended the running of the prescriptive period. HELD:The condition contained in the insurance policy that claims must be presented within one year after rejection is not merely a procedural requirement. The condition is an important matter, essential to a prompt settlement of claims against insurance companies, as it demands that insurance suits be brought by the insuredwhile the evidence as to the origin and cause of destruction have not yet disappeared. It is in the nature of a condition precedent to the liability of the insurer, or in other terms, a resolutory cause, the purpose of which is to terminate all liabilities in case the action is not filed by the insured within the period stipulated.The bringing of the action against the Paramount Surety & Insurance Company, theagent of the defendant Company cannot have any legal effect except that of notifying the agent of the claim. Beyond such notification, the filing of the actioncan serve no other purpose. There is no law giving any effect to such action upon the principal. Besides, there is no condition in the policy that the action must be filed against the agent, and this Court cannot by interpretation, extendthe clear scope of the agreement beyond what is agreed upon by the parties.Tthe rights of the parties flow from the contract of insurance, hence they are not bound by the statute of limitations nor by exemptions thereto. In the words of our own law, their contract is the law between the parties, and their agreement that an action on a claim denied by the insurer must be brought within one year from the denial, governs, not the rules on the prescription of actions. New Life v CA G.R. No. 94071 March 31, 1992 J. Regalado Facts: Julian Sy, owner of New Life, insured his building in 3 different insurance agencies for 350,000, 1,000,000, and 200,000. When his building and the goods inside burned down, he claimed for insurance indemnities, but these were rejected by the three companies for violation of policy conditions. Sy filed for 3 different suits in the trial court, where he won all suits against the insurance companies. The court of appeals reversed the decision of the trial court. Issue: Did the petitioner violate conditions 3 and 27 of the three insurance policies, thereby foreiting collection of indemnities? Held: Yes. Ratio: Condition 3. The insured shall give notice to the Company of any insurance or insurances already effected, or which may subsequently be effected, covering any of the property or properties consisting of stocks in trade, goods in process and/or inventories only hereby insured , and unless such notice be given and the particulars of such insurance or insurances be stated therein or endorsed on this policy pursuant to Section 50 of the Insurance Code, by or on behalf of the Company before the occurrence of any loss or damage, all benefits under this policy shall be deemed forfeited, provided however, that this condition shall not apply when the total insurance or insurances in force at the time of loss or damage not more than P200,000.00. Sy never disclosed co-insurance in the contracts he entered into with the three corporations. The insured is specifically required to disclose the insurance that he had contracted with other companies. Sy also contended that the insurance agents knew of the co-insurance. However, the theory of imputed knowledge, that the knowledge of the agent is presumed to be known by the principal, is not enough. When the words of the document are readily understandable by an ordinary reader, there is no need for construction anymore. The conformity of the insured to the terms of the policy is implied with his failure to disagree with the terms of the contract. Since Sy, was a businessman, it was incumbent upon him to read the contracts. Pioneer Insurance and Surety Corporation vs. Yap- The obvious purpose of the aforesaid requirement in the policy is to prevent over-insurance and thus avert the perpetration of fraud. The public, as well as the insurer, is interested in preventing the situation in which a fire would be profitable to the insured. Also, policy condition 15 was used. It stated: 15.. . . if any false declaration be made or used in support thereof, . . . all benefits under this Policy shall be forfeited . . .

As for condition number 27, the stipulation read: 27. Action or suit clause. If a claim be made and rejected and an action or suit be not commenced either in the Insurance Commission or any court of competent jurisdiction of notice of such rejection, or in case of arbitration taking place as provided herein, within twelve (12) months after due notice of the award made by the arbitrator or arbitrators or umpire, then the claim shall for all purposes be deemed to have been abandoned and shall not thereafter be recoverable hereunder. This is regarding Sys claim for one of the companies. Recovery was filed in court by petitioners only on J anuary 31, 1984, or after more than one (1) year had elapsed from petitioners' receipt of the insurers' letter of denial on November 29, 1982. This made it void. Paulino vs Capital Ins. & Surety Co. [105 Phil. 1315, May 15, 1959] Facts: Plaintiff Paulino secured a fire insurance policy issued by defendant Capital Insurance on Feb. 8, 1952. On April 30, 1952 the Plaintiff wrote the defendant requesting cancellation of the policy, which the latter received on May 10, 1952. The plaintiff did not return the policy nor de4manded for the return of the proportionate premium and neither did the defendant offer to breturn the premium. The property covered by the policy was destroyed by fire on aug. 15, 1952. The defendant refused to pay plaintiffs claim on the ground that the policy was cancelled as of May 10, 1952. Plaintiff contends in this appeal that her letter, dated April 30, 1952, was a mere request orb offer to cancel the policy and did not terminate the same since it was not accompanied by the surrender of the policy for cancellation, Issue: Whether the policy as effectively cancelled or not. Held: This case hinges on the interpretation of paragraph 10 of the policy, reading: This insurance may be terminated anytime at the request of the Insured, in which case the Company will retain the customary period rate for the time the policy has been enforced. This insurance may also at any time be terminated at the option of the Company, on notice to that effect being given to the insured, in which case the Company shall be liable to repay on demand a ratable proportion of the premium for the expired term from the date of cancellment Pursuant to this stipulation,the contract in question could be terminated at any time up on the unilateral act of either party. Whichever party exercised the option did not need the approval. Consent nor concurrence of the other thereto, That consent was given at the making of the contract. Moreover, pursuant to her letter, plaintiff considered the contract terminated upon receipt of said letter by the defendant (deade el recibo de la presente0, Decision of the lower court dismissing the4 action to recover the amount pf fire insurance policy is affirmed. (note: this ruling is deemed modigfied by Sec. 65} Malayan Ins. Co. Vs. Cruz Arnaldo [154 SCRA 672] FACTS: On June 7, 1981, the petitioner (hereinafter called (MICO) issued to the private respondent, Coronacion Pinca, Fire Insurance Policy No. F-001-17212 on her property for the amount of P14,000.00 effective July 22, 1981, until July 22, 1982 On October 15,1981, MICO allegedly cancelled the policy for non-payment, of the premium and sent the corresponding notice to Pinca.

On December 24, 1981, payment of the premium for Pinca was received by Domingo Adora, agent of MICO. On January 15, 1982, Adora remitted this payment to MICO,together with other payments. On January 18, 1982, Pinca's property was completely burned. On February 5, 1982, Pinca's payment was returned by MICO to Adora on the ground that her policy had been cancelled earlier. But Adora refused to accept it. In due time, Pinca made the requisite demands for payment, which MICO rejected. She then went to the Insurance Commission. It is because she was ultimately sustained by the public respondent that the petitioner has come to us for relief. ISSUE: Whether or not the cancellation of the policy by the insurance company was valid RULING: The Supreme Court ruled in favor of the insured. The above provision (Sec. 77 as invoked by the insurance company) is not applicable because payment of the premium was in fact eventually made in this case. Notably, the premium invoice issued to Pinca at the time of the delivery of the policy on June 7, 1981 was stamped "Payment Received" of the amount of P930.60 on "12-24-81" by Domingo Adora. This is important because it suggests an understanding between MICO and the insured that such payment could be made later, as agent Adora had assured Pinca. In any event, it is not denied that this payment was actually made by Pinca to Adora, who remitted the same to MICO. It is not disputed that the premium was actually paid by Pinca to Adora on December 24, 1981, who received it on behalf of MICO, to which it was remitted on January 15, 1982. What is questioned is the validity of Pinca's payment and of Adora's authority to receive it. MICO's acknowledgment of Adora as its agent defeats its contention that he was not authorized to receive the premium payment on its behalf. It is clearly provided in Section 306 of the Insurance Code. And it is a well-known principle under the law of agency that: Payment to an agent having authority to receive or collect payment is equivalent to payment to the principal himself; such payment is complete when the money delivered is into the agent's hands and is a discharge of the indebtedness owing to the principal. The policy could be cancelled on any of the supervening grounds enumerated in Article 64 (except "nonpayment of premium") provided the cancellation was made in accordance therewith and with Article 65. Section 64 reads as follows: SEC. 64. No policy of insurance other than life shall be cancelled by the insurer except upon prior notice thereof to the insured, and no notice of cancellation shall be effective unless it is based on the occurrence, after the effective date of the policy, of one or more of the following: (a) non-payment of premium; (b) conviction of a crime arising out of acts increasing the hazard insured against; (c) discovery of fraud or material misrepresentation; (d) discovery of willful, or reckless acts or commissions increasing the hazard insured against;

(e) physical changes in the property insured which result in the property becoming uninsurable; or (f) a determination by the Commissioner that the continuation of the policy would violate or would place the insurer in violation of this Code. As for the method of cancellation, Section 65 provides as follows: SEC. 65. All notices of cancellation mentioned in the preceding section shall be in writing, mailed or delivered to the named insured at the address shown in the policy, and shall state (a) which of the grounds set forth in section sixty-four is relied upon and (b) that, upon written request of the named insured, the insurer will furnish the facts on which the cancellation is based. A valid cancellation must, therefore, require concurrence of the following conditions: (1) There must be prior notice of cancellation to the insured; (2) The notice must be based on the occurrence, after the effective date of the policy, of one or more of the grounds mentioned; (3) The notice must be (a) in writing, (b) mailed, or delivered to the named insured, (c) at the address shown in the policy; (4) It must state (a) which of the grounds mentioned in Section 64 is relied upon and (b) that upon written request of the insured, the insurer will furnish the facts on which the cancellation is based. Thus, it behooved MICO's to make sure that the cancellation was actually sent to and received by the insured. No such proof in support thereto was presented by the insurance company. As it has not been shown that there was a valid cancellation of the policy, there was consequently no need to renew it but to pay the premium thereon. Payment was thus legally made on the original transaction and it could be, and was, validly received on behalf of the insurer by its agent Adora. Adora, incidentally, had not been informed of the cancellation either and saw no reason not to accept the said payment. Lastly, Loss and its amount may be determined on the basis of such proof as may be offered by the insured, which need not be of such persuasiveness as is required in judicial proceedings. 25 If, as in this case, the insured files notice and preliminary proof of loss and the insurer fails to specify to the former all the defects thereof and without unnecessary delay, all objections to notice and proof of loss are deemed waived under Section 90 of the Insurance Code. Ang Giok Chip v Springfield G.R. No. L-33637 December 31, 1931 J. Malcolm Facts: Ang insured his warehouse for the total value of Php 60,000. One of these, amounting to 10,000, was with Springfield Insurance Company. His warehouse burned down, then he attempted to recover 8,000 from Springfield for the indemnity. The insurance company interposed its defense on a rider in the policy in the form of Warranty F, fixing the amount of hazardous good that can be stored in a building to be covered by the insurance. They claimed that Ang violated the 3 percent limit by placing hazardous goods to as high as 39 percent of all the goods stored in the building. His suit to recover was granted by the trial court. Hence, this appeal. Issue: Whether a warranty referred to in the policy as forming part of the contract of insurance and in the form of a rider to the insurance policy, is null and void because not complying with the Philippine Insurance Act. Held: No. The warranty is valid. Petition dismissed. Ratio: The Insurance Act, Section 65, taken from California law, states:

"Every express warranty, made at or before the execution of a policy, must be contained in the policy itself, or in another instrument signed by the insured and referred to in the policy, as making a part of it." Warranty F, indemnifying for a value of Php 20,000 and pasted on the left margin of the policy stated: It is hereby declared and agreed that during the currency of this policy no hazardous goods be stored in the Building to which this insurance applies or in any building communicating therewith, provided, always, however, that the Insured be permitted to stored a small quantity of the hazardous goods specified below, but not exceeding in all 3 per cent of the total value of the whole of the goods or merchandise contained in said warehouse, viz; . . . . Also, the court stated a book that said, "any express warranty or condition is always a part of the policy, but, like any other part of an express contract, may be written in the margin, or contained in proposals or documents expressly referred to in the policy, and so made a part of it." It is well settled that a rider attached to a policy is a part of the contract, to the same extent and with like effect as it actually embodied therein. In the second place, it is equally well settled that an express warranty must appear upon the face of the policy, or be clearly incorporated therein and made a part thereof by explicit reference, or by words clearly evidencing such intention. The court concluded that Warranty F is contained in the policy itself, because by the contract of insurance agreed to by the parties it was made to be a part. It wasnt aseparate instrument agreed to by the parties. The receipt of the policy by the insured without objection binds him. It was his duty to read the policy and know its terms. He also never chose to accept a different policy by considering the earlier one as a mistake. Hence, the rider is valid. Young v Midland March 31, 1915 G.R. No. L-9370 J. Johnson

Facts: Young owned a candy and fruit store in Manila. Midland issued a policy for the payment of a premium of P60. The indemnity was 3,000 if the place was destroyed by fire. One clause claimed: Waranty B. It is hereby declared and agreed that during the pendency of this policy no hazardous goods stored or kept for sale, and no hazardous trade or process be carried on, in the building to which this insurance applies, or in any building connected therewith. Young then placed three boxes of fireworks. The plaintiff intended to use them for Chinese New Year, but the authorities prohibited the use. The bodega was destroyed by fire. Both of the parties agree that said fireworks come within the phrase hazardous goods, mentioned in said warranty B of the policy. But it was found out that the fireworks were found in a part of the building not destroyed by the fire, and that they in no way contributed to the fire. Issue: Whether or not the placing of said fireworks in the building insured is a violation of the terms of the contract of insurance and especially of warranty B Held: Yes. Petition dismissed. Ratio: Both the plaintiff and defendant agree that if they were hazardous goods, and if they were stored, then the act of the plaintiff was a violation of the terms of the contract of insurance and the defendant was justified in repudiating its liability. This leads us to a consideration of the meaning of the accord stored as used in said warranty B. Whether a particular article is stored or not must, in some degree, depend upon the intention of the parties. Nearly all of the cases cited by the lower court are cases where the article was being put to some reasonable and actual use, which might easily have been permitted by the terms of the policy, and within the intention of the parties, and excepted from the operation of the warranty, like the present. The author of the Century Dictionary defines the world store to be a deposit in a store or warehouse for preservation or safe keeping; o place in a warehouse or other place of deposit for safe keeping. Said definitions, of course, do not include a deposit in a store, in small quantities, for daily use. Daily use precludes the idea of a deposit for preservation or safe keeping, as well as a deposit for future consumption, or safe keeping.

The plaintiff makes no claim that he deposited them there with any other idea than for future use for future consumption. It seems clear to us that the hazardous goods in question were stored in the bodega, as that word is generally defined. That being true, suppose the defendant had made an examination of the premises, even in the absence of a fire, and had found the hazardous goods there, under the conditions above described, would it not have been justified, then and there, in declaring the policy null and of no effect by reason of a violation of its terms? The appellant argues, however, that in view of the fact that the storing of the fireworks on the premises of the insured did not contribute in any way to the damage occasioned by the fire, he should be permitted to recover that the storing of the hazardous goods in no way caused injury to the defendant company. The storing was a violation of the terms of the contract by virtue of the provisions of the policy itself, terminated the contractual relations. The plaintiff paid a premium based upon the risk at the time the policy was issued. Certainly it cannot be denied that the placing of the firecrackers in the building insured increased the risk. The plaintiff had not paid a premium based upon the increased risk, neither had the defendant issued a policy upon the theory of a different risk. The plaintiff was enjoying, if his contention may be allowed may be allowed, the benefits of an insurance policy upon one risk, whereas, as a matter of fact, it was issued upon an entirely different risk. The defendant had neither been paid nor had issues a policy to cover the increased risk. An increase of risk which is substantial and which is continued for a considerable period of time, is a direct and certain injury to the insurer, and changes the basis upon which the contract of insurance rests. Qua Chee Gan v. Law Union Rock - Breach of Warranty 98 PHIL 85 Facts: > Qua Chee Gan, a merchant, owned 4 warehouses in Albay which were used for the storage or copra and hemp in which the appelle deals with exclusively. > The warehouses together with the contents were insured with Law Union since 1937 and the loss made payable to PNB as mortgagee of the hemp and copra. > A fire of undetermined cause broke out in July 21, 1940 and lasted for almost 1 whole week. > Bodegas 1, 3, and 4 including the merchandise stored were destroyed completely. > Insured then informed insurer of the unfortunate event and submitted the corresponding fire claims, which were later reduced to P370T. > Insurer refused to pay claiming violations of the warranties and conditions, filing of fraudulent claims and that the fire had been deliberately caused by the insured. > Insured filed an action before CFI which rendered a decision in favor of the insured. Issues and Resolutions: (1) Whether or not the policies should be avoided for the reason that there was a breach of warranty. Under the Memorandum of Warranty, there should be no less than 1 hydrant for each 150 feet of external wall measurements of the compound, and since bodegas insured had an external wall per meter of 1640 feet, the insured should have 11 hydrants in the compound. But he only had 2. Even so, the insurer is barred by estoppel to claim violation of the fire hydrants warranty, because knowing that the number of hydrants it demanded never existed from the very beginning, appellant nevertheless issued the policies subject to such warranty and received the corresponding premiums. The insurance company was aware, even before the policies were issued, that in the premises there were only 2 hydrants and 2 others were owned by the Municipality, contrary to the requirements of the warranties in question. It should be close to conniving at fraud upon the insured to allow the insurer to claim now as void the policies it issued to the insured, without warning him of the fatal defect, of which the insurer was informed, and after it had misled the insured into believing that the policies were effective. Accdg to American Jurisprudence: It is a well-settled rule that the insurer at the time of the issuance of a policy has the knowledge of existing facts, which if insisted on, would invalidate the contract from its very inception, such knowledge constitutes a waiver of conditions in the contract inconsistent with known facts, and the insurer is stopped thereafter from asserting the breach of such conditions. The reason for the rule is: To allow a company to accept ones money for a policy of insurance which it knows to be void and of no effect, though it knows as it must

that the insured believes it to be valid and binding is so contrary to the dictates of honesty and fair dealing, as so closely related to positive fraud, as to be abhorrent to fair-minded men. It would be to allow the company to treat the policy as valid long enough to get the premium on it, and leave it at liberty to repudiate it the next moment. Moreover, taking into account the well-known rule that ambiguities or obscurities must strictly be interpreted against the party that cause them, the memorandum of warranty invoked by the insurer bars the latter from questioning the existence of the appliances called for, since its initial expression the undernoted appliances for the extinction of fire being kept on the premises insured hereby.. admits of the interpretation as an admission of the existence of such appliances which insurer cannot now contradict, should the parole evidence apply. (2) Whether or not the insured violated the hemp warranty provision against the storage of gasoline since insured admitted there were 36 cans of gasoline in Bodega 2 which was a separate structure and not affected by the fire. It is well to note that gasoline is not specifically mentioned among the prohibited articles listed in the so-called hemp warranty. The clause relied upon by the insurer speaks of oils. Ordinarily, oils mean lubricants and not gasoline or kerosene. Here again, by reason of the exclusive control of the insurance company over the terms of the contract, the ambiguity must be held strictly against the insurer and liberally in favor of the insured, specially to avoid a forfeiture. Furthermore, the gasoline kept was only incidental to the insureds business. It is a well settled rule that keeping of inflammable oils in the premises though prohibited by the policy does NOT void it if such keeping is incidental to the business. Also, the hemp warranty forbade the storage only in the building to which the insurance applies, and/or in any building communicating therewith; and it is undisputed that no gasoline was stored in the burnt bodegas and that Bodega No. 2 which was where the gasoline was found stood isolated from the other bodegas. Gen. Insurance & Surety Corp v. NG Hua - Misrepresentation 106 PHIL 1117 Facts: > In 1952, General issued a fire policy to Ng Hua to cover the contents of the Central Pomade Factory owned by him. > There was a provision in the policy that should there be any insurance already effected or to be subsequently procured, the insured shall give notice to the insurer. > Ng Hua declared that there was non. The very next day, the building and the goods stored therein burned. > Subsequently, the claim of Ng Hua for the proceeds was denied by General since it discovered that Ng Hua had obtained an insurance from General Indemnity for the same goods and for the same period of time. Issue: Whether or not General Insurance can refuse to pay the proceeds. Held: Yes. Violation of the statement which is to be considered a warranty entitles the insurer to rescind the contract of insurance. Such misrepresentation is fatal. Bautista v. Capital Insurance - Insurance Policy 1 CA Rep. 228 Facts: > In 1952, a contract of insurance was entered by the parties, upon PIlar Bautistas house. > The policy described the building as occupied as dwelling only. > There was a stipulation to the effect that any misrepresentation of material fact or misdescription of the property shall render the insurer not liable for its loss. > Before the policy was issued however, Manuel Leyson, Bautistas lessee, subleased the ground floor to ONg, who used it as a factory for the manufacture of shoes. A month later, rubber heels, soles and canvass were stored therein. > Subsequently, the house was destroyed by fire. Bautista filed her claims with Capital Insurance, but the latter denied her claim on the ground of breach of warranty. > Bautista said that the statement occupied as dwelling only was not hers, but of the insurance agent, and that the policy was in English (which she did not understand) and was never read to her.

Issue: Whether or not Capital may rescind the contract. Held: It can. Bautista was bound to know the contents of the policy in accepting it. In the absence of fraud, she is presumed to know the contents of the contract and to have assented to them. Failure to read the policy is negligence, and the insured is regarded as having assumed the risk of the falsity or misstatements of its contents. Insurance Case Digest: Philippine Phoenix Surety & Insurance Co. v. Woodworks Inc (1979) G.R. No. L-25317 August 6, 1979 Lessons Applicable: Estoppel and credit extension (Insurance) Laws Applicable: Section 77 of the Insurance Code FACTS: July 21, 1960: Woodworks, Inc. was issued a fire policy for its building machinery and equipment by Philippine Phoenix Surety & Insurance Co. for P500K covering July 21, 1960 to July 21, 1961. Woodworks did not pay the premium totalling to P10,593.36. April 19, 1961: It was alleged that Woodworks notified Philippine Phoenix the cancellation of the Policy so Philippine Phoenix credited P3,110.25 for the unexpired period of 94 days and demanded in writing the payment of P7,483.11 Woodworks refused stating that it need not pay premium "because the Insurer did not stand liable for any indemnity during the period the premiums were not paid." Philippine Phoenix filed with the CFI to recover its earned premium of P7,483.11 Woodworks: to pay the premium after the issuance of the policy put an end to the insurance contract and rendered the policy unenforceable CFI: favored Philippine Phoenix ISSUE: W/N there was a valid insurance contract despite no premium payment was paid HELD: NO. Reversed Policy provides for pre-payment of premium. To constitute an extension of credit there must be a clear and express agreement therefor and there nust be acceptance of the extension - none here Since the premium had not been paid, the policy must be deemed to have lapsed. failure to make a payment of a premium or assessment at the time provided for, the policy shall become void or forfeited, or the obligation of the insurer shall cease, or words to like effect, because the contract so prescribes and because such a stipulation is a material and essential part of the contract. This is true, for instance, in the case of life, health and accident, fire and hail insurance policies Explicit in the Policy itself is plaintiff's agreement to indemnify defendant for loss by fire only "after payment of premium" Compliance by the insured with the terms of the contract is a condition precedent to the right of recovery. The burden is on an insured to keep a policy in force by the payment of premiums, rather than on the insurer to exert every effort to prevent the insured from allowing a policy to elapse through a failure to make premium payments. Insurance Case Digest: Makati Tuscany v. CA (1992) G.R. No. 95546 November 6, 1992 Lessons Applicable: Installments and partial payment (Insurance) Grounds on Return of Premium: No exposed to peril insured against (Insurance) FACTS: Early 1982: American Home Assurance Co. (AHAC), represented by American International Underwriters (Phils.), Inc., issued in favor of Makati Tuscany Condominium Corporation (Tuscany) on the latter's building and premises, for a period beginning 1 March 1982 and ending 1 March 1983, with a total premium of P466,103.05.

Premium were paid on installments on: March 12 1982 May 20 1982 June 21 1982 November 16 1982 February 10 1983: AHAC replaced and renewed the previous policy, for a term covering 1 March 1983 to 1 March 1984 premium of P466,103.05 was again paid on installments on: April 13 1983 July 13 1983 August 3 1983 September 9 1983 November 21 1983 January 20 1984: policy was again renewed for the period March 1 1984 to March 1 1985 Tuscany only paid two installment payments February 6 1984 for P52k June 6 1984 for P100k AHAC filed an action to recover the unpaid balance of P314,103.05 RTC: dismissed the complaint While it is true that the receipts issued to the defendant contained the aforementioned reservations, it is equally true that payment of the premiums of the three aforementioned policies (being sought to be refunded) were made during the lifetime or term of said policies, hence, it could not be said, inspite of the reservations, that no risk attached under the policies counterclaim for refund is not justified CA: ordered Tuscany to pay premiums when due is ordinarily as indivisible obligation to pay the entire premium; insurance contract became valid and binding upon payment of the first premium ISSUE: 1. W/N payment by installment of the premiums due on an insurance policy invalidates the contract of insurance on the basis of: Sec. 77 of the Insurance Code, no contract of insurance is valid and binding unless the premium thereof has been paid, notwithstanding any agreement to the contrary. As a consequence, petitioner seeks a refund of all premium payments made on the alleged invalid insurance policies. 2. W/N there is risk attached to the insurance so it cannot be refunded HELD: 1. NO Section 77 merely precludes the parties from stipulating that the policy is valid even if premiums are not paid, but does not expressly prohibit an agreement granting credit extension, and such an agreement is not contrary to morals, good customs, public order or public policy At the very least, both parties should be deemed in estoppel to question the arrangement they have voluntarily accepted. It paid the initial installment and thereafter made staggered payments resulting in full payment of the 1982 and 1983 insurance policies. For the 1984 policy, petitioner paid 2 installments although it refused to pay the balance. appearing that they actually intended to make 3 insurance contracts valid 2. NO. where the risk is entire and the contract is indivisible, the insured is not entitled to a refund of the premiums paid if the insurer was exposed to the risk insured for any period, however brief or momentary Areola v CA G.R. No. 95641 September 22, 1994 J. Romero Facts:

Prudential Guarantee cancelled Areolas personal accident insurance on the grounds that the latter failed to pay his premiums 7 months after issuing the policy. Areola was supposed to pay the total amount of P1,609.65 which included the premium of P1,470.00, documentary stamp of P110.25 and 2% premium tax of P29.40. The statement of account had a stipulation not considering it a receipt. It also reminded the customer to ask for a receipt after payment. There was also a stipulation calling for a demand for a provisional receipt after payment to an agent. A provisional receipt was sent to petitioner telling him that the provisional receipt would be confirmed by an official one. The company then cancelled the policy for non-payment of premiums. After being surprised, Areola confronted a company agent and demanded an official receipt. The latter told him that it was a mistake, but never gave him an official receipt. Areola sent a letter demanding that he be reinstated or he would file for damages if his demand was not met. The company then told him that his payments werent in full yet. The company replied to Areola by telling him that there was reason to believe that no payment has been made since no official receipt was issued. The company then told him that they would still hold him under the policy. The company then confirmed that he paid the premium and that they would extend the policy by one year. Thereby, the company offered to reinstate same policy it had previously cancelled and even proposed to extend its lifetime on finding that the cancellation was erroneous and that the premiums were paid in full by petitioner-insured but were not remitted by the company's branch manager, Mr. Malapit. However, they were too late for Areola already filed an action for breach of contract in the trial court. The companys defense lay in rectifying its omission; hence, there was no breach of contract. The court ruled in favor of Areola and asked Prudential to pay 250,000 pesos in moral and exemplary damages. The court held that the company was in bad faith in cancelling the policy. Had the insured met an accident at that time, he wouldnt be covered by the policy. This ruling was challenged on appeal by respondent insurance company, denying bad faith in unilaterally cancelling the policy. The AC absolved Prudential on the grounds that it was not motivated by negligence, malice or bad faith in cancelling subject policy. Rather, the cancellation of the insurance policy was based on what the existing records showed. The court even added that the errant manager who didnt remit the profits was forced to resign. Areola then filed for a petition in the Supreme Court. Issue: 1. Did the erroneous act of cancelling subject insurance policy entitle petitioner-insured to payment of damages? 2. Did the subsequent act of reinstating the wrongfully cancelled insurance policy by respondent insurance company, in an effort to rectify such error, obliterate whatever liability for damages it may have to bear, thus absolving it? Held: Yes. No. Petition granted. Ratio: 1. Petitioner alleged that the managers misappropriation of his premium payments is the proximate cause of the cancellation of the insurance policy. Subsequent reinstatement could not possibly absolve respondent insurance company from liability, due to the breach of contract. He contended that damage had already been done. Prudential averred that the equitable relief sought by petitioner-insured was granted to the filing of the complaint, petitioner-insured is left without a cause of action. Reinstatement effectively restored petitioner-insured to all his rights under the policy. The court held that Malapit's fraudulent act of misappropriating the premiums paid by petitioner-insured is directly imputable to respondent insurance company. A corporation, such as respondent insurance company, acts solely thru its employees. The latters' acts are considered as its own. Malapit represented its interest and acted in its behalf. His act of receiving the premiums collected is well within the province of his authority. Thus, his receipt of said premiums is receipt by private respondent insurance company who, by provision of law is bound by the acts of its agent. Article 1910 thus reads: Art. 1910. The principal must comply with all the obligations which the agent may have contracted within the scope of his authority. As for any obligation wherein the agent has exceeded his power, the principal is not bound except when he ratifies it expressly or tacitly. Malapit's failure to remit the premiums he received cannot constitute a defense for private respondent insurance company; no exoneration from liability could result therefrom. The fact that private respondent insurance company was itself defrauded due to the anomalies that took place does not free the same from its obligation to petitioner Areola. As held in Prudential Bank v. Court of Appeals

A bank is liable for wrongful acts of its officers done in the interests of the bank or in the course of dealings of the officers in their representative capacity but not for acts outside the scope of their authority. Accordingly, a banking corporation is liable to innocent third persons where the representation is made in the course of its business by an agent acting within the general scope of his authority even though the agent is secretly abusing his authority and attempting to perpetrate a fraud upon his principal or some other person. Prudential is liable for damages for the fraudulent acts committed by Malapit. Reinstating the insurance policy can not obliterate the injury inflicted. A contract of insurance creates reciprocal obligations for both insurer and insured. Reciprocal obligations are those which arise from the same cause and in which each party is both a debtor and a creditor of the other, such that the obligation of one is dependent upon the obligation of the other. 2. Due to the agreement to enter into a contract of insurance where Prudential promised to extend protection to petitioner-insured against the risk insured, there was a debtor creditor relation ship between the two parties. Under Article 1191, the injured party is given a choice between fulfillment or rescission of the obligation in case one of the obligors fails to comply with what is incumbent upon him. However, said article entitles the injured party to payment of damages, regardless of whether he demands fulfillment or rescission of the obligation. The damages would be nominal because the insurance company took steps to rectify the contract . There was also no actual or substantial damage inflicted. Nominal damages are "recoverable where a legal right is technically violated and must be vindicated against an invasion that has produced no actual present loss of any kind, or where there has been a breach of contract and no substantial injury or actual damages whatsoever have been or can be shown. Tibay v CA G.R. No. 119655. May 24, 1996 J. Bellosillo: Facts: Fortune Life issued a fire insurance Policy to Tibay on her two-storey residential building at Zobel Street, Makati City. The insurance was for P600,000.00 covering the period from January 23, 1987 to January 23, 1988. On January 23 1987, Tibay only paid P600.00 of 3,000 peso premium and left a balance. The insured building was completely destroyed by fire. Tibay then paid the balance. On the same day, she filed a claim on the policy. Her claim was accordingly referred to the adjuster, Goodwill, which immediately wrote Violeta requesting her to furnish it with the necessary documents for the investigation and processing of her claim. Petitioner complied, and she signed a non-waiver agreement. Fortune denied the claim for violation of the Insurance Code. Tibay sued for damages in the amount of P600,000.00 representing the total coverage of the policy. The trial court ruled for petitioners and made fortune liable for the total value of the insured building and personal properties. The Court of Appeals reversed the court by removing liability from Fortune after returning the premium. Hence this petition for review. The petitioner contended that Fortune remained liable under the subject fire insurance policy in spite of the failure of petitioners to pay their premium in full. Issue: May a fire insurance policy be valid, binding and enforceable upon mere partial payment of premium? Held: No. Petition dismissed. Ratio: The pertinent provisions read: 2. This policy including any renewal thereof and/or any endorsement thereon is not in force until the premium has been fully paid to and duly receipted by the Company in the manner provided herein. This policy shall be deemed effective, valid and binding upon the Company only when the premiums therefor have actually been paid in full and duly acknowledged in a receipt signed by any authorized official of the company Where the premium has only been partially paid and the balance paid only after the peril insured against has occurred, the insurance contract did not take effect and the insured cannot collect at all on the policy. The Insurance Code which says that no policy or contract of insurance issued by an insurance company is valid and binding unless and until the premium has been paid. What does unless and until the premium thereof has been paid mean? Escosura v. San Miguel- the legislative practice was to interpret with pay in accordance to the intention of distinguish between full and partial payment, where the modifying term is used.

Petitioners used Philippine Phoenix v. Woodworks, where partial payment of the premium made the policy effective during the whole period of the policy. The SC didnt consider the 1967 Phoenix case as persuasive due to the different factual scenario. In Makati Tuscany v CA, the parties mutually agreed that the premiums could be paid in installments, hence, this Court refused to invalidate the insurance policy. Nothing in Article 77 of the Code suggested that the parties may not agree to allow payment of the premiums in installment, or to consider the contract as valid and binding upon payment of the first premium. Phoenix and Tuscany demonstrated the waiver of prepayment in full by the insurer. In this case however, there was no waiver. There was a stipulation that the policy wasnt in force until the premium has been fully paid and receipted. There was no juridical tie of indemnification from the fractional payment of premium. The insurance contract itself expressly provided that the policy would be effective only when the premium was paid in full. Verily, it is elemental law that the payment of premium is requisite to keep the policy of insurance in force. If the premium is not paid in the manner prescribed in the policy as intended by the parties the policy is ineffective. Partial payment even when accepted as a partial payment will not keep the policy alive. South Sea v CA stipulated 2 exceptions to the requirement of payment of the entire premium as a prerequisite to the validity of the insurance contract. These are when in case the insurance coverage relates to life or insurance when a grace period applies, and when the insurer makes a written acknowledgment of the receipt of premium to be conclusive evidence of payment. Hence, in the absence of clear waiver of prepayment in full by the insurer, the insured cannot collect on the proceeds of the policy. The terms of the insurance policy constitute the measure of the insurers liability. In the absence of statutory prohibition to the contrary, insurance companies have the same rights as individuals to limit their liability and to impose whatever conditions they deem best upon their obligations not inconsistent with public policy. Dissent: J. Vitug All the calculations of the company are based on the hypothesis of prompt payments. They not only calculate on the receipt of the premiums when due, but on the compounding interest upon them. It is on this basis that they are enabled to offer assurance at the favorable rates they do. The failure of appellants to fully pay their premium prevented the contract of insurance from becoming binding an Fortune. This series of acts is tainted with misrepresentation and violates the uberrimae fidae principle of insurance contracts. Tibay had entered into a "Non-Waiver Agreement" with the adjuster which permitted Fortune to claim non-payment of premium as a defense. The law neither requires, nor measures the strength of the vinculum juris by any specific amount of premium payment. Payment on the premium, partly or in full, is made by the insured which the insurer accepts. In fine, it is either that a juridical tie exists (by such payment) or that it is not extant at all (by an absence thereof). Once the juridical relation comes into being, the full efficacy follows. This is a partially performed contract. The non-payment of the balance shouldnt result in an automatic cancellation of the contract; otherwise, the right to decide the effectivity of the contract would become potestative. Instead, the parties should be able to demand from each other the performance of whatever obligations they had assumed or, if desired, sue timely for the rescission of the contract. In the meanwhile, the contract endures, and an occurrence of the risk insured riggers the insurer's liability. Also, legal compensation arises where insurer's liability to the insured would simply be reduced by the balance of the premium. It must here be noted that the insured had made, and the insurer had accepted partial premium payment on the policy weeks before the risk insured against took place. An insurance is an aleatory contract effective upon its perfection although the occurrence of a condition or event may later dictate the demandability of certain obligations. Fortunes stipulation that insurance shall not "be . . . in force until the premium has been fully paid," and that it "shall be deemed effective, valid and binding upon the company only when the premiums therefor have actually been paid in full and duly acknowledged," override the efficaciousness of the insurance contract despite the payment and acceptance. Article 78 of the Insurance Code An acknowledgment in a policy or contract of insurance of the receipt of premium is conclusive evidence of its payment, so far as to make the policy binding, notwithstanding any stipulation therein that it shall not be binding until the premium is actually paid

Even if a portion was paid in the premium, the insurance coverage becomes effective and binding, any stipulation in the policy to the contrary notwithstanding. American Home v Chua G.R. No. 130421. June 28, 1999 C.J. Davide Facts: Chua obtained from American Home a fire insurance covering the stock-in-trade of his business. The insurance was due to expire on March 25, 1990. On April 5, 1990, Chua issued a check for P2,983.50 to American Homes agent, James Uy, as pa yment for the renewal of the policy. The official receipt was issued on April 10. In turn, the latter a renewal certificate. A new insurance policy was issued where petitioner undertook to indemnify respondent for any damage or loss arising from fire up to P200,000 March 20, 1990 to March 25, 1991. On April 6, 1990, the business was completely razed by fire. Total loss was estimated between P4,000,000 and P5,000,000. Respondent filed an insurance claim with petitioner and four other co-insurers, namely, Pioneer Insurance, Prudential Guarantee, Filipino Merchants and Domestic Insurance. Petitioner refused to honor the claim hence, the respondent filed an action in the trial court. American Home claimed there was no existing contract because respondent did not pay the premium. Even with a contract, they contended that he was ineligible bacue of his fraudulent tax returns, his failure to establish the actual loss and his failure to notify to petitioner of any insurance already effected. The trial court ruled in favor of respondent because the respondent paid by way of check a day before the fire occurred and that the other insurance companies promptly paid the claims. American homes was made to pay 750,000 in damages. The Court of Appeals found that respondents claim was substantially proved and petitioners unjustified refusal to pay the claim entitled respondent to the award of damages. American Home filed the petition reiterating its stand that there was no existing insurance contract between the parties. It invoked Section 77 of the Insurance Code, which provides that no policy or contract of insurance issued by an insurance company is valid and binding unless and until the premium thereof has been paid and the case of Arce v. Capital Insurance that until the premium is paid there is no insurance. Issues: 1. Whether there was a valid payment of premium, considering that respondents check was cashed after the occurrence of the fire 2. Whether respondent violated the policy by his submission of fraudulent documents and non-disclosure of the other existing insurance contracts 3. Whether respondent is entitled to the award of damages. Held: Yes. No. Yes, but not all damages valid. Petition granted. Damages modified. Ratio: 1. The trial court found, as affirmed by the Court of Appeals, that there was a valid check payment by respondent to petitioner. The court respected this. The renewal certificate issued to respondent contained the acknowledgment that premium had been paid. In the instant case, the best evidence of such authority is the fact that petitioner accepted the check and issued the official receipt for the payment. It is, as well, bound by its agents acknowledgment of receipt of payment. Section 78 of the Insurance Code explicitly provides: An acknowledgment in a policy or contract of insurance of the receipt of premium is conclusive evidence of its payment, so far as to make the policy binding, notwithstanding any stipulation therein that it shall not be binding until the premium is actually paid. 2. Submission of the alleged fraudulent documents pertained to respondents income tax returns for 1987 to 1989. Respondent, however, presented a BIR certification that he had paid the proper taxes for the said years. Since this is a question of fact, the finding is conclusive. Ordinarily, where the insurance policy specifies as a condition the disclosure of existing co-insurers, non-disclosure is a violation that entitles the insurer to avoid the policy. The purpose for the inclusion of this clause is to prevent an increase in the moral hazard. The relevant provision is Section 75, which provides that: A policy may declare that a violation of specified provisions thereof shall avoid it, otherwise the breach of an immaterial provision does not avoid the policy.

Respondent acquired several co-insurers and he failed to disclose this information to petitioner. Nonetheless, petitioner is estopped from invoking this argument due to the loss adjusters admission of previous knowledge of the co-insurers. It cannot be said that petitioner was deceived by respondent by the latters non -disclosure of the other insurance contracts when petitioner actually had prior knowledge thereof. The loss adjuster, being an employee of petitioner, is deemed a representative of the latter whose awareness of the other insurance contracts binds petitioner. 3. Petitioner is liable to pay the loss. But there is merit in petitioners grievance against the damages and attorneys fees awarded. There was no basis for an award for loss of profit. This cannot be shouldered by petitioner whose obligation is limited to the object of insurance. There was no fraud to justify moral damages. Exemplary damages cant be awarded because the defendant never acted in a reckless manner to claim insurance. Attorneys fees cant be recovered as part of damages because no premium should be placed on the right to litigate. GREAT PACIFIC LIFE INSURANCE CORPORATION vs. THE HON. COURT OF APPEALS and TEODORO CORTEZ G.R. No. L-57308 April 23, 1990 FACTS: Teodoro Cortez applied for a 20-year endowment policy for P30,000 upon the solicitation of Margarita Siega, an underwriter of GREPALIFE. His application, with the requisite medical examination, was thereafter accepted and approved by GREPALIFE, and thereafter, Endowment Policy No. 221944 was issued in his name. It was released for delivery on January 24, 1973, and was actually delivered to him by the underwriter, Mrs. Siega on January 25, 1973. The effective date indicated on the face of the policy in question was December 25, 1972. The annual premium was P1,416.60. Mrs. Siega assured him that the first premium may be paid within the grace period of thirty (30) days from date of delivery of the policy. As such, the first premium was paid by him in three instalments: o P400 was paid on 2/5/1973 evidenced by temporary receipt # 19422 (issued by Siega) which was later confirmed by OR no. 43543 dtd 3/6/1973 (issued by GREPALIFE) o P350 was paid on 2/17/1973 evidenced by temporary receipt # 19448 (issued by Siega) which was later confirmed by OR no. 43559 dtd 3/28/1973 (issued by GREPALIFE) o P666.60 was paid on 2/21/1973 evidenced by temporary receipt # 19702 (issued by Siega) which was later confirmed by OR no. 43563 dtd 3/28/1973 (issued by GREPALIFE) June 1, 1973 GREPALIFE advised plaintiff that his policy was not in force. And to make it so, GREALIFE asked Cortez to remit the balance of P1,015.60 to complete his initial annual premium due on December 15, 1972 and to see Dr. Felipe V. Remollo for another medical exam at his own expense. Cortez then informed GREPALIFE that he was cancelling the policy and demanded the return of the premium he paid plus damages. This demand was ignored. As his demands were ignored, Cortez filed a complaint for damages with the CFI, praying for the refund of the insurance premium he paid, plus damages. Sept 9, 1977 CFI rendered judgment in favor of the plaintiff GREPALIFE appealed to the CA and the latter rendered a decision affirming the CFIs decision. MR filed by GREPALIFE denied. ISSUE: WON Cortez is entitled to a refund of his premium. HELD: Yes.

RATIO: Factual findings of the CA: o Record shows that premiums were fully paid on February 21, 1973 or within the grace period. This being so, the policy was already enforceable. The official receipts issued by the GREPALIFE's agent can only mean that it ratified the act of Mrs. Margarita Siega in giving

Cortez a grace period of 30 days from January 25, 1973 within which to pay the annual premium. When GREPALIFE advised Cortezn June 1, 1973, four months after he had paid the first premium, that his policy had never been in force, and that he must pay another premium and undergo another medical examination to make the policy effective, GREPALIFE committed a serious breach of the contract of insurance. GREPALIFE should have informed Cortez of the deadline for paying the first premium before or at least upon delivery of the policy to him, so he could have complied with what was needful and would not have been misled into believing that his life and his family were protected by the policy, when actually they were not. And, if the premium paid by Cortez was unacceptable for being late, it was the company's duty to return it. By accepting his premiums without giving him the corresponding protection, the company acted in bad faith. Sections 79, 81 and 82 of P.D. 612 of the Insurance Code of 1978 provide when the insured is entitled to the return of premium paid. SECTION 79. A person insured is entitled to a return of premium, as follows: (a) To the whole premium, if no part of his interest in the thing insured be exposed to any of the perils insured against. (b) Where the insure is made for a definite period of time and the insured surrenders his policy, to such portion of the premium as corresponds with the unexpired time, at a pro rata rate, unless a short period rate has been agreed upon and appears on the face of the policy, after deducting from the whole premium any claim for loss or damage under the policy which has previously accrued: Provided, That no holder of a life insurance policy may avail himself of the privileges of this paragraph without sufficient causes as otherwise provided by law. SECTION 81. A person insured is entitled to a return of the premium when the contract is voidable on account of the fraud or misrepresentation of the insurer or of his agent or on account of facts the existence of which the insured was ignorant without his fault; or when, by any default of the insured other than actual fraud, the insurer never incurred any liability under the policy. SECTION 82. In case of an over-insurance by several insurers, the insured is entitled to a ratable return of the premium, proportioned to the amount by which the aggregate sum insured in all the policies exceeds the insurable value of the thing at risk. Since his policy was in fact inoperative or ineffectual from the beginning, the company was never at risk, hence, it is not entitled to keep the premium. PETITION DENIED. PETITIONER ORDERED TO REFUND TO PRIVATE RESPONDENT PREMIUMS IT PAID WITH LEGAL INTEREST OF 6% P.A.

Paris-Manila Perfume Co. v. Phoenix Assurance Co.(1926) Lessons Applicable: Loss, the immediate cause of which was the peril insured against, if the proximate cause thereof was NOT excepted in the contract (Insurance) Facts: May 22, 1924: A fire insurance policy was issued by Phoenix Assurance Company, Limited to Messrs. ParisManila Perfumery Co. (Peter Johnson, Prop.) for P13,000 also insured with other insurance companies for P1,200 and P5,000 respectively July 4, 1924: The Perfumery was burned unknown of the cause totalling a loss of P38.025.56 Phoenix refused to pay nor to appoint an arbitrator stating that the policy did not cover any loss or damage occasioned by explosion and stating that the claim was fraudulent RTC: ordered Phoenix to pay P13,000 Phoenix appealed The insurance policy contains: Unless otherwise expressly stated in the policy the insurance does not cover (h) Loss or damage occasioned by the explosion; but loss or damage by explosion of gas for illuminating or domestic purposes in a building in which gas is not generated and which does not form a part of any gas works, will be deemed to be loss by fire within the meaning of this policy.

ISSUE: W/N Phoenix should be liable for the loss because there was no explosion which is an exemption from the policy HELD: YES. If it be a fact that the fire resulted from an explosion that fact, if proven, would be a complete defense, the burden of the proof of that fact is upon the defendant, and upon that point, there is a failure of proof lower court found as a fact that there was no fraud in the insurance, and that the value of the property destroyed by the fire was more than the amount of the insurance.
THE EAST FURNITURE INC vs. THE GLOBE & RUTGERS FIRE INSURANCE CO. OF NEW YORK [G.R. No. L-35848 November 22, 1932] Facts: Plaintiff is a duly registered partnership engaged in the sale of furniture; that the defendant is a company engaged in the insurance business. Plaintiff insured against fire the articles existing in its establishment. On March 2, 1929, a fire broke out in plaintiff's establishment, as a result of which the insured articles therein found were destroyed by the fire. Defendants defenses are:(1) that the fire in question was of intentional origin; (2) that the claims of loss presented by the plaintiff were false and fraudulent; (3) that the furniture in question had been mortgaged by the plaintiff to the Manila Finance and Discount Corporation, so that at the time of the fire the plaintiff was not the only party interested therein, contrary to the representations made in its claims of loss; and (4) that the plaintiff violated one of the conditions of the policies by refusing to furnish the defendants with a physical inventory of the contents of its store at the time of the fire. By agreement of the parties the three cases were tried jointly who after the trial found that the claims presented by the plaintiff were notoriously fraudulent, and, accordingly, sustained defendant's second special defense and dismissed the complaint in each of the three cases. Hence, this petition. Issue: W/N fire was of intentional origin W/N the claim of loss were fraudulent Ruling: Fire of Intentional Origin We are thus led to the conclusion that defendants' first special defense is well founded that the fire in question was of intentional origin and was caused with the connivance of the plaintiff. Neither the interest of the justice nor public policy would be promoted by an omission of the courts to expose and condemn incendiarism once the same is established by competent evidence. It would tend to encourage rather than suppress that great public menace if the courts do not expose the crime to public condemnation when the evidence in a case like the present shows that it has really been committed. Fraudulent claim of loss We may also consider the damage caused by the fire in relation with defendant's second special defense that plaintiff's claims of loss were false and fraudulent. To each of the proofs of loss which the plaintiff presented to the respective insurance companies four days after the fire was attached an inventory of the furniture claimed to have been in the building at the time of the fire. This inventory contains 506 pieces of furniture and 3,700 board feet of lumber of the alleged total value of P52,061.99. This amount was the total loss claimed to have been suffered by the plaintiff, although we note that in its complaints in these cases amended it is conceded that some furniture of the value of about P5,000 was saved.

Regardless of any difference of opinion as to the value of the insured furniture and the extent of the damage caused thereto by the fire in question, the fact that the insured only had approximately 202 pieces of furniture in the building at the time of the fire and sought to compel the insurance companies to pay for 506 pieces conclusively shows that its claim was not honestly conceived. The trial court's conclusion that said claim is notoriously fraudulent, is correct. Condition 12 of each of the insurance policies sued upon provides that "if the claim be in any respect fraudulent, or if any false declaration be made or used in support thereof, or if any fraudulent means or devices are used by the Insured or anyone acting on his behalf to obtain any benefit under this policy; or, if the loss or damage be occasioned by the wilful act, or with the connivance of the Insured, all benefit under this policy shall be forfeited."

FGU Insurance Corporation v. CA (2005) G.R.No. 137775 March 31, 2005 Lessons Applicable: Loss caused by negligence of the insured (Insurance) FACTS: Anco Enterprises Company (ANCO), a partnership between Ang Gui and Co To, was engaged in the shipping business operating two common carriers M/T ANCO tugboat D/B Lucio barge - no engine of its own, it could not maneuver by itself and had to be towed by a tugboat for it to move from one place to another. September 23 1979: San Miguel Corporation (SMC) shipped from Mandaue City, Cebu, on board the D/B Lucio, for towage by M/T ANCO: 25,000 cases Pale Pilsen and 350 cases Cerveza Negra - consignee SMCs Beer Marketing Division (BMD) Estancia Beer Sales Office, Estancia, Iloilo 15,000 cases Pale Pilsen and 200 cases Cerveza Negra - consignee SMCs BMD-San Jose Beer Sales Office, San Jose, Antique September 30, 1979: D/B Lucio was towed by the M/T ANCO arrived and M/T ANCO left the barge immediately The clouds were dark and the waves were big so SMCs District Sales Supervisor, Fernando Macabuag, requested ANCOs representative to transfer the barge to a safer pl ace but it refused so around the midnight, the barge sunk along with 29,210 cases of Pale Pilsen and 500 cases of Cerveza Negra totalling to P1,346,197 When SMC claimed against ANCO it stated that they agreed that it would not be liable for any losses or damages resulting to the cargoes by reason of fortuitous event and it was agreed to be insured with FGU for 20,000 cases or P858,500 ANCO filed against FGU FGU alleged that ANCO and SMC failed to exercise ordinary diligence or the diligence of a good father of the family in the care and supervision of the cargoes RTC: ANCO liable to SMC and FGU liable for 53% of the lost cargoes CA affirmed ISSUE: W/N FGU should be exempted from liability to ANCO for the lost cargoes because of a fortuitous event and negligence of ANCO HELD: YES. Affirmed with modification. Third-party complainant is dismissed. Art. 1733. Common carriers, from the nature of their business and for reasons of public policy are bound to observe extraordinary diligence in the vigilance over the goods and for the safety of the passengers transported by them, according to all the circumstances of each case. Such extraordinary diligence in vigilance over the goods is further expressed in Articles 1734, 1735, and 1745 Nos. 5, 6, and 7 . . .

Art. 1734. Common carriers are responsible for the loss, destruction, or deterioration of the goods, unless the same is due to any of the following causes only:

(1) . . .

Flood, storm, earthquake, lightning, or other natural disaster or calamity;

Art. 1739. In order that the common carrier may be exempted from responsibility, the natural disaster must have been the proximate and only cause of the loss. However, the common carrier must exercise due diligence to prevent or minimize loss before, during and after the occurrence of flood, storm, or other natural disaster in order that the common carrier may be exempted from liability for the loss, destruction, or deterioration of the goods . . . Caso fortuito or force majeure extraordinary events not foreseeable or avoidable, events that could not be foreseen, or which though foreseen, were inevitable not enough that the event should not have been foreseen or anticipated, as is commonly believed but it must be one impossible to foresee or to avoid - not in this case other vessels in the port of San Jose, Antique, managed to transfer to another place To be exempted from responsibility, the natural disaster should have been the proximate and only cause of the loss. There must have been no contributory negligence on the part of the common carrier. there was blatant negligence on the part of M/T ANCOs crewmembers, first in leaving the engine -less barge D/B Lucio at the mercy of the storm without the assistance of the tugboat, and again in failing to heed the request of SMCs representatives to have the barge transferred to a safer place When evidence show that the insureds negligence or recklessness is so gross as to be s ufficient to constitute a willful act, the insurer must be exonerated. ANCOs employees is of such gross character that it amounts to a wrongful act which must exonerate FGU from liability under the insurance contract both the D/B Lucio and the M/T ANCO were blatantly negligent Bachrach v. British American Insurance Co. - Insurance Proceeds 17 PHIL 555 Facts: > Bachrach insured properties of its general furniture shop with British. The properties were subsequently destroyed by fire. > Bachrach claims from the insurance company. The claim was denied on the ff grounds: o The policy was allegedly forfeited because the insured stored varnishes and paints within the premises; o Insured stored gasoline in the building; and o Bachrach executed a chattel mortgage on the properties insured without the consent of the insured. Issue: Whether or not Bachrach can claim the proceeds of the policy. Held: Yes. The policy was NOT forfeited due to the strong paints and varnishes. There was no express provision pertaining to it and these paints and varnishes are incidental to the business of the insured to keep the furniture in a saleable condition. The gasoline stored within the premises was in the reservoir of the car and thus does not violate any provision in the policy. There is no express prohibition against the execution of a chattel mortgage on the property insured. VDA. DE FERNANDEZ v. NATIONAL LIFE INSURANCE CO OF THE PHILS 105 PHIL 59 ENDENCIA; January 27, 1959 NATURE

Appeal from CFI decision applying the Ballantyne scale of values upon the proceeds of life insurance taken and maturing during the Japanese occupation but claimed after liberation FACTS - National Life Insurance Company (NLIC) insured J. Fernandezs li fe for P10,000 upon his payment of P444 from July 15, 1944 to July 14, 1945 - The insured died on November 2, 1944, while the policy was in force - After more than 7 years, in 1952, Atty de la Torre, representing the benficiaries of the policy, informed the company that Fernandez had died in 1944, and claimed the proceeds of the policy. The company said that the status of the policies issued during the Japanese occupation was still pending consideration before the courts. NLIC said that because the policy matured upon the insureds death in November, 1944 , they should compute the value of their claim under the Ballantyne scale of values (which would amount only to P500) - beneficiaries commenced suit, and the lower court sustained the stand of the company, dismissed the complaint. - beneficiaries maintain that the obligation of the company to pay accrued not upon the death of Fernandez, but only upon the receipt and approval by the company, on proof of death of the insured, which was in 1954. The policy reads: National Life Insurance Company of the Philippine hereby agrees to pay at its Home Office, Manila, Ten Thousand Pesos to Juan D. Fernandez (hereinafter called the insured) on the 15th day of July, 1964, if the Insured is living and this Policy is in force, or upon receipt and approved at its Office of due proofs of the title of the claimant and of the prior death of the Insured while this Policy is in force to Teresa Duat Vda. De Fernandez, Maria T. and Manuela Fernandez, mother and sisters respectively of the Insured (Hereinafter called the Beneficiary) subject to the right of the Insured to change the beneficiary as stated on the second page of this Policy. - The above stipulation is apparently based on Sec. 91-A of the Insurance Law which provides as follows: The proceeds of a life insurance policy shall be paid immediately upon maturity of the policy, unless such proceeds are made payable in installments or a as an annuity, in which case the installments or annuities shall be paid as they become due: Provided, however, That in case of a policy maturing by the death of the insured , the proceeds thereof shall be paid within sixty days after presentation of the claim and filing of the proof of the death of the insured. Refused to pay the claim within the time prescribed herein will entitle the beneficiary to collect interest on the proceeds of the policy for the duration of the delay at the rate of six per centum per annum, unless such failure or refusal to pay is based on the ground that the claim is fraudulent . . . . - Based on the foregoing provision of law and the aforequoted stipulation as well as on the allegation that the filing of proof of death by the beneficiaries is a condition precedent of the demandability of the obligation of the insurer to pay the proceeds, appellants claim that they should be paid P10,000 in Philippine currency and not under the Ballantyne scale of values. ISSUE WON the policy matured upon the death of the insured HELD YES Ratio In life insurance, the policy matures either upon the expiration of the term set forth therein, or upon his death occuring at any time prior to the expiration of such stipulated term, in which case, the proceeds are payable to his beneficiaries within sixty days after their filing of proof of death. Reasoning - The sixty day period fixed by law within which to pay the proceeds after presentation of proof of death Is merely procedural in nature, evidently to determine the exact amount to be paid and the interest thereon to which the beneficiaries may be entitled to collect in case of unwarranted refusal of the company to pay, and also to enable the insurer to verify or check on the fact of death which it may even validly waive. It is the happening of the suspensive condition of death that renders a life policy matured, and not ht efiling of proof of death which, as above stated, is merely procedural. The insured having died during the Japanese occupation, the proceeds of his policy should be adjusted accordingly, for The rule is already settled that where a debtor could have paid his obligation at any time during the Japanese occupation, payment after liberation must be adjusted in accordance with the Ballantyne schedule (De Asis vs. Agdamag, among other cases). (Collaboration is defined as the acts of working together in a joint project. Disposition Judgment affirmed

Pacific Banking Corporation vs. CA & Oriental Assurance [GR. No. L-41014 28 November 1988] Facts: An open Fire Policy issued to Paramount Shirt Manufacturing for Php61,000 on the following: stocks, materils, supplies, furniture, fixture, machinery, equipment contained on the 1st to 3rd floors. Insurance is for a year starting 21 OCTOBER 1964. Paramount Shirt is debtor of Pacific Banking amounting to Php800,000. Goods in policy were held in trust by Paramount for Pacific under thrust receipts. Fire broke out on 4 January 1964. Pacific sent letter of demand to Oriental. Insurance Adjuster of Oriental notified Pacific to submit proof of loss pursuant to Policy Condition 11. Pacific did not accede but asked Insurance Adjuster to verify records form Bureau of Customs. Pacific filed for sum of money against Oriental. Oriental alleged that Pacific prematurely filed a suit, for neither filing a formal claim over loss pursuant to policy nor submitting any proof of loss. Trial court decided in favor of Pacific. Decision based on technicality. The defense of lack of proof of loss and defects were raised for the 1st time. (On presentation of evidences by Pacific, it was revealed there was violation of Condition No.3, there were undeclared co-insurances under same property Wellington, Empire, Asian. The only declared co-insurances were Malayan, South Sea, and Victory) CA reversed decision. Concealment of other co-insurances is a misrepresentation and can easily be fraud. Issues: (1) Whether or not unrevealed con-insurances is a violation of Policy Condition No.3 (2) Whether or not there was premature filing of action Held: (1) Yes. Policy Condition 3 provides that the insured must give notice of any insurance already in effect or subsequently be in effect covering same property being insured. Failure to do so, the policy shall be forfeited. Failure to reveal before the loss of the 3 other insurances is a clear misrepresentation or a false declaration. The material fact was asked for but was not revealed. Representations of facts are the foundations of the contract. Pacific itself provided for the evidences in trial court that proved existence of misrepresentation. (2) Yes. Policy Condition 11 is a sine qua non requirement for maintaining action. It requires that documents necessary to prove and estimate the loss should be included with notice of loss. Pacific failed to submit formal claim of loss with supporting documents but shifted the burden to the insurance company. Failing to submit claim is failure for insurance company to reject claim. Thus, a lack of cause of action to file suit. Furthermore, the mortgage clause in the policy specifically provides that the policy is invalidated by reasons of FRAUD, MISREPRESENTATION and FRAUD. Concealment can easily be fraud or misrepresentation. The insured PARAMOUNT is not entitled to proceeds. Moreso, Pacific as indorsee of policy is not entitled. Pacific Timber v. CA 112 SCRA 199 Facts: > On March 13, 1963, Pacific secured temporary insurance from the Workemens Insurance Co. for its exportation of logs to Japan. Workmen issued on said date Cover Note 1010 insuring said cargo. > The regular marine policies were issued by the company in favor of Pacific on Apr 2, 1963. The 2 marine policies bore the number 53H01032 and 53H01033. > After the issuance of the cover note but BEFORE the issuance of the 2 policies, some of the logs intended to be exported were lost due to a typhoon.

> Pacific filed its claim with the company, but the latter refused, contending that said loss may not be considered as covered under the cover note because such became null and void by virtue of the issuance of the marine policies. Issue: Whether or not the cover not was without consideration, thus null and void. Held: It was with consideration. SC upheld Pacifics contention that said cover not was with consideration. The fact that no separate premium was paid on the cover note before the loss was insured against occurred does not militate against the validity of Pacifics contention, for no such premium could have been paid, since by the nature of the cover note, it did not contain, as all cover notes do not contain, particulars of the shipment that would serve as basis for the computation of the premiums. As a logical consequence, no separate premiums are required to be paid on a cover note. If the note is to be treated as a separate policy instead of integrating it to the regular policies subsequently issued, its purpose would be meaningless for it is in a real sense a contract, not a mere application. PHILIPPINE CHARTER INSURANCE CORPORATION VS. CHEMOIL LIGHTERAGE HITE GOLD CORPORATION G.R. No. 136888. June 29, 2005 Facts: Philippine Charter Insurance Corporation is a domestic corporation engaged in the business of non-life insurance. Respondent Chemoil Lighterage Corporation is also a domestic corporation engaged in the transport of goods. On 24 January 1991, Samkyung Chemical Company, Ltd., based in South Korea, shipped 62.06 metric tons of the liquid chemical DIOCTYL PHTHALATE (DOP) on board MT TACHIBANA which was valued at US$90,201.57 and another 436.70 metric tons of DOP valued at US$634,724.89 to the Philippines. The consignee was Plastic Group Phils., Inc. in Manila. PGP insured the cargo with Philippine Charter Insurance Corporation against all risks. The insurance was under Marine Policies No. MRN-30721[5] dated 06 February 1991. Marine Endorsement No. 2786[7] dated 11 May 1991 was attached and formed part of MRN-30721, amending the latters insured value to P24,667,422.03, and reduced the premium accordingly. The ocean tanker MT TACHIBANA unloaded the cargo to the tanker barge, which shall transport the same to Del Pan Bridge in Pasig River and haul it by land to PGPs storage tanks in Calamba, Laguna. Upon inspection by PGP, the samples taken from the shipment showed discoloration demonstrating that it was damaged. PGP then sent a letter where it formally made an insurance claim for the loss it sustained. Petitioner requested the GIT Insurance Adjusters, Inc. (GIT), to conduct a Quantity and Condition Survey of the shipment which issued a report stating that DOP samples taken were discolored. Inspection of cargo tanks showed manhole covers of ballast tanks ceilings loosely secured and that the rubber gaskets of the manhole covers of the ballast tanks re-acted to the chemical causing shrinkage thus, loosening the covers and cargo ingress. Petitioner paid PGP the full and final payment for the loss and issued a Subrogation Receipt. Meanwhile, PGP paid the respondent the as full payment for the latters services. On 15 July 1991, an action for damages was instituted by the petitioner-insurer against respondent-carrier before the RTC, Br.16, City of Manila. Respondent filed an answer which admitted that it undertook to transport the shipment, but alleged that before the DOP was loaded into its barge, the representative of PGP, Adjustment Standard Corporation, inspected it and found the same clean, dry, and fit for loading, thus accepted the cargo without any protest or notice. As carrier, no fault and negligence can be attributed against respondent as it exercised extraordinary diligence in handling the cargo. After due hearing, the trial court rendered a Decision in favor of plaintiff. On appeal, the Court of Appeals promulgated its Decision reversing the trial court. A petition for review on certiorar[ was filed by the petitioner with this Court. Issues: 1. Whether or not the Notice of Claim was filed within the required period. 2.Whether or not the damage to the cargo was due to the fault or negligence of the respondent. Held: Article 366 of the Code of Commerce has profound application in the case at bar, which provides that; Within twenty-four hours following the receipt of the merchandise a claim may be made against the carrier on account of damage or average found upon opening the packages, provided that the indications of the damage or

average giving rise to the claim cannot be ascertained from the exterior of said packages, in which case said claim shall only be admitted at the time of the receipt of the packages. After the periods mentioned have elapsed, or after the transportation charges have been paid, no claim whatsoever shall be admitted against the carrier with regard to the condition in which the goods transported were delivered. As to the first issue, the petitioner contends that the notice of contamination was given by PGP employee, to Ms. Abastillas, at the time of the delivery of the cargo, and therefore, within the required period. The respondent, however, claims that the supposed notice given by PGP over the telephone was denied by Ms. Abastillas. The Court of Appeals declared:that a telephone call made to defendant-company could constitute substantial compliance with the requirement of notice. However, it must be pointed out that compliance with the period for filing notice is an essential part of the requirement, i.e.. immediately if the damage is apparent, or otherwise within twenty-four hours from receipt of the goods, the clear import being that prompt examination of the goods must be made to ascertain damage if this is not immediately apparent. We have examined the evidence, and We are unable to find any proof of compliance with the required period, which is fatal to the accrual of the right of action against the carrier.[27] Nothing in the trial courts decision stated that the notice of claim was relayed or filed with the respondent-carrier immediately or within a period of twenty-four hours from the time the goods were received. The Court of Appeals made the same finding. Having examined the entire records of the case, we cannot find a shred of evidence that will precisely and ultimately point to the conclusion that the notice of claim was timely relayed or filed. The requirement that a notice of claim should be filed within the period stated by Article 366 of the Code of Commerce is not an empty or worthless proviso. The object sought to be attained by the requirement of the submission of claims in pursuance of this article is to compel the consignee of goods entrusted to a carrier to make prompt demand for settlement of alleged damages suffered by the goods while in transport, so that the carrier will be enabled to verify all such claims at the time of delivery or within twenty-four hours thereafter, and if necessary fix responsibility and secure evidence as to the nature and extent of the alleged damages to the goods while the matter is still fresh in the minds of the parties. The filing of a claim with the carrier within the time limitation therefore actually constitutes a condition precedent to the accrual of a right of action against a carrier for loss of, or damage to, the goods. The shipper or consignee must allege and prove the fulfillment of the condition. If it fails to do so, no right of action against the carrier can accrue in favor of the former. The aforementioned requirement is a reasonable condition precedent; it does not constitute a limitation of action.[31] We do not believe so. As discussed at length above, there is no evidence to confirm that the notice of claim was filed within the period provided for under Article 366 of the Code of Commerce. Petitioners contention proceeds from a false presupposition that the notice of claim was timely filed. Considering that we have resolved the first issue in the negative, it is therefore unnecessary to make a resolution on the second issue. Pioneer v Yap G.R. No. L-36232 December 19, 1974 J. Fernandez Facts: Respondent Oliva Yap was the owner of a store in a two-storey building where she sold shopping bags and footwear. Chua Soon Poon, her son-in-law, was in charge of the store. Yap took out a Fire Insurance Policy No. 4216 from Pioneer Insurance with a value of P25,000.00 covering her stocks, office furniture, fixtures and fittings. Among the conditions in the policy executed by the parties are the following: unless such notice be given and the particulars of such insurance or insurances be stated in, or endorsed on this Policy by or on behalf of the Company before the occurrence of any loss or damage, all benefits under this Policy shall be forfeited Any false declaration or breach or this condition will render this policy null and void. Another insurance policy for P20,000.00 issued by Great American covering the same properties. The endorsement recognized co-insurance by Northwest for the same value. Oliva Yap took out another fire insurance policy for P20,000.00 covering the same properties from the Federal Insurance Company, Inc., which was procured without notice to and the written consent of Pioneer. A fire broke out in the building, and the store was burned. Yap filed an insurance claim, but the same was denied for a breach. Oliva Yap filed a case for payment of the face value of her fire insurance policy. The insurance company refused to pay because she never informed Pioneer of another insurer. The trial court decided in favor of Yap. The CA affirmed.

Issue: Whether or not petitioner should be absolved from liability on the Pioneeer policy on account of any violation of the co-insurance clause Held: No. Petition dismissed. Ratio: There was a violation. The insurance policy for P20,000.00 issued by the Great American, ceased to be recognized by them as a co-insurance policy. The endorsement shows the clear intention of the parties to recognize on the date the endorsement was made, the existence of only one co-insurance, the Northwest one. The finding of the Court of Appeals that the Great American Insurance policy was substituted by the Federal Insurance policy is indeed contrary to said stipulation. Other insurance without the consent of Pioneer would avoid the contract. It required no affirmative act of election on the part of the company to make operative the clause avoiding the contract, wherever the specified conditions should occur. Its obligations ceased, unless, being informed of the fact, it consented to the additional insurance. The validity of a clause in a fire insurance policy to the effect that the procurement of additional insurance without the consent of the insurer renders the policy void is in American jurisprudence. Milwaukee Mechanids' Lumber Co., vs. Gibson- "The rule in this state and practically all of the states is to the effect that a clause in a policy to the effect that the procurement of additional insurance without the consent of the insurer renders the policy void is a valid provision. In this jurisdiction, General Insurance & Surety Corporation vs. Ng Hua- The annotation then, must be deemed to be a warranty that the property was not insured by any other policy. Violation thereof entitled the insurer to rescind. Furthermore, even if the annotations were overlooked the defendant insurer would still be free from liability because there is no question that the policy issued by General Indemnity has not been stated in nor endorsed on Policy No. 471 of defendant. The obvious purpose of the aforesaid requirement in the policy is to prevent over-insurance and thus avert the perpetration of fraud where a fire would be profitable to the insured. Geagonia v CA G.R. No. 114427 February 6, 1995 Facts: Geagonia, owner of a store, obtained from Country Bankers fire insurance policy for P100,000.00. The 1 year policy and covered thestock trading of dry goods. The policy noted the requirement that "3. The insured shall give notice to the Company of any insurance or insurances already effected, or which may subsequently be effected, covering any of the property or properties consisting of stocks in trade, goods in process and/or inventories only hereby insured, and unless notice be given and the particulars of such insurance or insurances be stated therein or endorsed in this policy pursuant to Section 50 of the Insurance Code, by or on behalf of the Company before the occurrence of any loss or damage, all benefits under this policy shall be deemed forfeited, provided however, that this condition shall not apply when the total insurance or insurances in force at the time of the loss or damage is not more than P200,000.00." The petitioners stocks were destroyed by fire. He then filed a claim which was subsequently denied because the petitioners stocks were covered by two other fire insurance policies for Php 200,00 0 issued by PFIC. The basis of the private respondent's denial was the petitioner's alleged violation of Condition 3 of the policy. Geagonia then filed a complaint against the private respondent in the Insurance Commission for the recovery of P100,000.00 under fire insurance policy and damages. He claimed that he knew the existence of the other two policies. But, he said that he had no knowledge of the provision in the private respondent's policy requiring him to inform it of the prior policies and this requirement was not mentioned to him by the private respondent's agent. The Insurance Commission found that the petitioner did not violate Condition 3 as he had no knowledge of the existence of the two fire insurance policies obtained from the PFIC; that it was Cebu Tesing Textiles w/c procured the PFIC policies w/o informing him or securing his consent; and that Cebu Tesing Textile, as his creditor, had insurable interest on the stocks. The Insurance Commission then ordered the respondent company to pay complainant the sum of P100,000.00 with interest and attorneys fees. CA reversed the decision of the Insurance Commission because it found that the petitioner knew of the existence of the two other policies issued by the PFIC.

Issues: 1. WON the petitioner had not disclosed the two insurance policies when he obtained the fire insurance and thereby violated Condition 3 of the policy. 2. WON he is prohibited from recovering Held: Yes. No. Petition Granted Ratio: 1. The court agreed with the CA that the petitioner knew of the prior policies issued by the PFIC. His letter of 18 January 1991 to the private respondent conclusively proves this knowledge. His testimony to the contrary before the Insurance Commissioner and which the latter relied upon cannot prevail over a written admission made ante litem motam. It was, indeed, incredible that he did not know about the prior policies since these policies were not new or original. 2. Stated differently, provisions, conditions or exceptions in policies which tend to work a forfeiture of insurance policies should be construed most strictly against those for whose benefits they are inserted, and most favorably toward those against whom they are intended to operate. With these principles in mind, Condition 3 of the subject policy is not totally free from ambiguity and must be meticulously analyzed. Such analysis leads us to conclude that (a) the prohibition applies only to double insurance, and (b) the nullity of the policy shall only be to the extent exceeding P200,000.00 of the total policies obtained. Furthermore, by stating within Condition 3 itself that such condition shall not apply if the total insurance in force at the time of loss does not exceed P200,000.00, the private respondent was amenable to assume a co-insurer's liability up to a loss not exceeding P200,000.00. What it had in mind was to discourage over-insurance. Indeed, the rationale behind the incorporation of "other insurance" clause in fire policies is to prevent over-insurance and thus avert the perpetration of fraud. When a property owner obtains insurance policies from two or more insurers in a total amount that exceeds the property's value, the insured may have an inducement to destroy the property for the purpose of collecting the insurance. The public as well as the insurer is interested in preventing a situation in which a fire would be profitable to the insured. Equitable Insurance and Casualty Insurance Co. vs. Rural Insurance and Surety Co. Facts: Equitable reinsured with Rural the stock covered by the fire insurance policy issued by Equitable to Messrs. Jaen Bermers Cooperative Marketing Association, Inc. The stocks were burned. A statement of account covering the share of the loss assumed by Rural was sent by Equitable. Despite repeated demands Rural refused to pay. Equitable likewise reinsured with Rural the stocks of Electric and Lamp Supplies covered by fire insurance policies. The stocks were burned. Rural refused to pay its share of loss. Hence this complaint for recovery was filed. Rural moved for its dismissal alleging that there is no cause of action. Invoking Article VIII of the Reinsurance Agreement, it alleged that the matter should be brought to the board of arbitrators before the court action could take place. Motion to dismiss denied. The RTC ruled in favor of Equitable. The appeal to CA was brought to the SC. Issue: Whether or not Equitable has a cause of action against Rural. Held: Yes. It is true that paragraph (Article VIII) of said Reciprocal Facultative Reinsurance Agreement required that 'in the event of any question arising as to the meaning of, or any way connected with or relating to this Agreement, whether before or after its termination, the parties shall endeavor to arrive at a satisfactory compromise by amicable settlement rather than by court action'; and that the dispute should be referred to the decision of two arbitrators and umpire, as provided, therein. However, in this particular case, there is absolutely no dispute between the two parties, because in the stipulation of facts, the defendant has admitted that plaintiff has paid its liability to the insured as per its fire insurance policies specified in the two causes of action of the complaint. Defendant has, likewise, admitted its liability as reinsurer under the Reciprocal Facultative Reinsurance Agreement to pay to the plaintiff its proportional shares, the amounts of which are not disputed. Indeed, according to the complaint as admitted by the defendant, statements of account as to the amounts of its share as reinsurer and, for all that appears, said defendant has never questioned the correctness of said amounts. The appellants claim that the court erred in failing to rule that in a facultative obligation the right to choose an alternative remedy lies only with the debtor, who in this case is Rural was held untenable. The term "facultative" is so used in reinsurance contracts merely to define the right of the reinsurer to accept or not to accept participation in

the risk insured. But once the share is accepted, as it was in the case at bar, the obligation is absolute and the liability assumed thereunder can be discharged by payment of the share of the losses. There is no alternative nor substitute prestation. Fieldman v Asian Surety Facts Asian Surety, the ceding company and Fieldman Insurance as the reinsurer entered into a reinsurance agreement. The treaties were to take effect from certain dates and were to be in force until cancelled by either party upon previous notice to the other party (3 months prior to the cancellation taking effect). The cancellation will take effect at the end of the year, i.e., Dec. 31. Fieldmens gave several notices of its desire to be relieved from all participation in the treaties, but with no reply from Asian. Subsequently, one of the risks reinsured with Fieldmens under cessions issued in favor of GSIS became a liability when the insured property was burned on februar 16, 1962. The policy was effected in July and was supposed to end the same month next year (1962). Fieldmen is claiming that the reinsurance contract ended on December 31 1961, thus even if the GSISs insurance will only expire July 1962, they cannot be held liable as they rescinded the reinsurance contract already. Issue. WON Fieldmen is liable to pay the fire reinsurance policy covering damages incurred by GSIS ceded to it by Asian surety. Held. Yes Ratio. The cancellation did not have the effect of terminating the liability of Fieldmens as reinsurer with respect to policies or cessions issued prior to the termination of the principal reinsurance contracts. Where the reinsurance contracts contain provisions which expressly provide the continuing effectivity of policies ceded under them for reinsurance, such cessions continue to be in force until their respective dates of expiration notwithstanding the cancellation of the contracts themselves.

ARTEX DEVELOPMENT CO INC v. WELLINGTON INSURANCE CO INC 51 SCRA 352 TEEHANKEE; June 27, 1973 FACTS - Wellington Insurance Co. Inc. insured for P24,346,509.00 the buildings, stocks and machinery of plaintiff Artex Development Co. Inc. against loss or damage by fire or lighting upon payment of the plaintiff of the corresponding premiums; that said properties were insured for an additional sum of P883,034.00; that defendant insured plaintiff against business interruption (use and occupancy) for P5,200,000.00; Wellington entered into a contract of reinsurance with Alexander and Alexander, Inc. of New York. USA. - The buildings, stocks and machineries of plaintiffs spinning department were burned. - Notice of the loss and damage was given the defendant; that as per report of the adjusters, the total property loss of the plaintiff was the sum of P10,106,554.40 and the total business interruption loss was P3,000,000.00; - That defendant has paid to the plaintiff the sum of P6,481,870.07 of the property loss suffered by plaintiff and P1,864,134.08 on its business interruption loss, leaving a balance of P3,624,683.43 and P1,748,460.00, respectively. - The counsel for Artex filed a Manifestation saying that in view of the Deeds of Discharge and Collateral Agreement, the only remaining liability subject of litigation shall be the proportion of the loss reinsured with or through Alexander and Alexander, Inc. of New York, USA, namely, P397,813.00. - The document recited further that Artex acknowledges receipt of the sum of P3.6M paid by the insurer in full and final settlement of all or any claims of Artex against its insurer. It discharges its insurer from all actions, proceedings, claims, demands, costs and expenses in respect thereof.

- With regard the balance unpaid, Wellington contends that Artex should have been directed against the reinsurers to cover the liability and not against Wellington. ISSUE WON the insured (Artex) has a cause of action against the reinsurer HELD NO - Unless there is a specific grant in, or assignment of, the reinsurance contract in favor of the insured or a manifest intention of the contracting parties to the reinsurance contract to grant such benefit or favor to the insured, the insured, not being privy to the reinsurance contract, has no cause of action against the reinsurer. It is expressly provided in Section 91 the Insurance Act 1 that "(T)he original insured has no interest in a contract of insurance." IVOR ROBERT DAYTON GIBSON, petitioner, vs. HON. PEDRO A. REVILLA, in his official capacity as Presiding Judge of Branch XIII, Court of First Instance of Rizal, and LEPANTO CONSOLIDATED MINING COMPANY, respondents. G.R. No. L-41432 July 30, 1979 FACTS: Lepanto Consolidated Mining Company filed a complaint against Malayan Insurance Company, Inc. The civil suit thus instituted by Lepanto against Malayan was founded on the fact that Malayan issued a Marine Open Policy covering all shipments of copper, gold, and silver concentrates in bulk from Poro, San Fernando, La Union to Tacoma, Washington or to other places in the United States. Thereafter, Malayan obtained reinsurance abroad through Sedgwick, Collins & Co., Limited, a London insurance brokerage. The Memorandum of Insurance issued by Sedgwick to Malayan listed three groups of underwriters or reinsurers Lloyds 62.808%, Companies (I.L.U.) 34.705%, Other companies 2.487%. At the top of the list of underwriting members of Lloyds is Syndicate No. 448, assuming 2.48% of the risk assumed by the reinsurer, which syndicate number petitioner Ivor Robert Dayton Gibson claims to be himself. Petitioner then filed a motion to intervene as defendant, which motion was denied by the lower court. ISSUE: WON THE LOWER COURT COMMITTED, REVERSIBLE ERROR IN REFUSING THE INTERVENTION OF THE PETITIONER IN THE SUIT BETWEEN LEPANTO AND MALAYAN COMPANIES. HELD: No. The respondent Judge committed no error of law in denying petitioners Motion to Intervene and neither has he abused his discretion in his denial of petitioners Motion for Intervention. We agree with the holding of the respondent court that since movant Ivor Robert Dayton Gibson appears to be only one of several re-insurers of the risks and liabilities assumed by Malayan Insurance Company, Inc., it is highly probable that other re-insurers may likewise intervene. If petitioner is allowed to intervene, We hold that there is good and sufficient basis for the Court a quo to declare that the trial between Lepanto and Malayan would be definitely disrupted and would certainly unduly delay the proceedings between the parties especially at the stage where Lepanto had already rested its case and that the issue would also be compounded as more parties and more matters will have to be litigated. In other words, the Courts discretion is justified and reasonable. We also hold that respondent Judge committed no reversible error in further sustaining the fourth ground of Lepantos Opposition to the Motion to Intervene that the rights, if any, of petitioner are not prejudiced by the present suit and will be fully protected in a separate action against him and his co-insurers by Malayan. Petitioners contention that he has to pay once Malayan is finally adjudged to pay Lepanto because of the very nature of a contract of reinsurance and considering that the re-insurer is obliged to pay as may be paid thereon (referring to the original policies), although this is subject to other stipulations and conditions of the reinsurance contract, is without merit. The general rule in the law of reinsurance is that the reinsurer is entitled to avail itself of every defense which the re-insured (which is Malayan) might urge in an action by the person originally insured (which is Lepanto). As to the effect of the clause to pay as may be paid thereon contained in petitioners re-insurance contract, Arnould, on the Law of Marine Insurance and Average, 13 th Ed., Vol. 1, Section 327, p. 315, states the rule, this: It has been decided that this clause does not preclude the reinsurer from insisting upon proper proof that a loss strictly within the terms of the original policy has taken place. This clause does not enable the original underwriter to recover from his reinsurer to an extent beyond the subscription of the latter. Wherefore, in view of the foregoing, the petition is hereby dismissed. No costs.

La Razon Social "Go Tiaoco y Hermanos" vs. Union Insurance Society of Canton Ltd. [GR 13983, 1 September 1919] Facts: A cargo of rice belonging to the Go Tiaoco Brothers, was transported in the early days of May, 1915, on the steamship Hondagua from the port of Saigon to Cebu. On discharging the rice from one of the compartments in the after hold, upon arrival at Cebu, it was discovered that 1,473 sacks had been damaged by sea water. The loss so resulting to the owners of rice, after proper deduction had been made for the portion saved, was P3,875. The policy of insurance, covering the shipment, was signed upon a form long in use among companies engaged in maritime insurance. It purports to insure the cargo from the following among other risks: "Perils . . . of the seas, men, of war, fire, enemies, pirates, rovers, thieves, .jettisons, . . . barratry of the master and mariners, and of all other perils, losses, and misfortunes that have or shall come to the hurt, detriment, or damage of the said goods and merchandise or any part thereof." It was found out that the drain pipe which served as a discharge from the water closet passed down through the compartment where the rice in question was stowed and thence out to sea through the wall of the compartment, which was a part of the wall of the ship. The joint or elbow where the pipe changed its direction was of cast iron; and in course of time it had become corroded and abraded until a longitudinal opening had appeared in the pipe about one inch in length. This hole had been in existence before the voyage was begun, and an attempt had been made to repair it by filling with cement and bolting over it a strip of iron. The effect of loading the boat was to submerge the vent, or orifice, of the pipe until it was about 18 inches or 2 feet below the level of the sea. As a consequence the sea water rose in the pipe. Navigation under these conditions resulted in the washing out of the cement-filling from the action of the sea water, thus permitting the continued flow of the salt water into the compartment of rice. An action on a policy of marine insurance issued by the Union Insurance Society of Canton, Ltd., upon the cargo of rice belonging to the Go Tiaoco Brothers was filed. The trial court found that the inflow of the sea water during the voyage was due to a defect in one of the drain pipes of the ship and concluded that the loss was not covered by the policy of insurance. Judgment was accordingly entered in favor of Union Insurance and Go Tiaoco Brothers appealed. Issue 1: Whether perils of the sea includes entrance of water into the ships hold through a defective pipe. Held 1: NO. It is determined that the words "all other perils, losses, and misfortunes" are to be interpreted as covering risks which are of like kind (ejusdem generis) with the particular risks which are enumerated in the preceding part of the same clause of the contract. According to the ordinary rules of construction these words must be interpreted with reference to the words which immediately precede them. They were no doubt inserted in order to prevent disputes founded on nice distinctions. Their office is to cover in terms whatever may be within the spirit of the cases previously enumerated, and so they have a greater or less effect as a narrower or broader view is taken of those cases. For example, if the expression "perils of the seas" is given its widest sense the general words have little or no effect as applied to that case. If on the other hand that expression is to receive a limited construction and loss by perils of the seas is to be confined to loss ex marine tempestatis discrimine, the general words become most important. But still, when they first became the subject of judicial construction, they have always been held or assumed to be restricted to cases "akin to" or "resembling" or "of the same kind as" those specially mentioned. I see no reason for departing from this settled rule. In marine insurance it is above all things necessary to abide by settled rules and to avoid anything like novel refinements or a new departure. It must be considered to be settled, furthermore, that a loss which, in the ordinary course of events, results from the natural and inevitable action of the sea, from the ordinary wear and tear of the ship, or from the negligent failure of the ship's owner to provide the vessel with proper equipment to convey the cargo under ordinary conditions,

is not a peril of the sea. Such a loss is rather due to what has been aptly called the "peril of the ship." The insurer undertakes to insure against perils of the sea and similar perils, not against perils of the ship. There must, in order to make the insurer liable, be "some casualty, something which could not be foreseen as one of the necessary incidents of the adventure. The purpose of the policy is to secure an indemnity against accidents which may happen, not against events which must happen." Herein, the entrance of the sea water into the ship's hold through the defective pipe already described was not due to any accident which happened during the voyage, but to the failure of the ship's owner properly to repair a defect of the existence of which he was apprised. The loss was therefore more analogous to that which directly results from simple unseaworthiness than to that which results from perils of the sea. Issue 2: Whether there is an implied warranty on the seaworthy of the vessel in every marine insurance contract. Held 2: YES. It is universally accepted that in every contract of insurance upon anything which is the subject of marine insurance, a warranty is implied that the ship shall be seaworthy at the time of the inception of the voyage. This rule is accepted in our own Insurance Law (Act No. 2427, sec. 106). It is also well settled that a ship which is seaworthy for the purpose of insurance upon the ship may yet be unseaworthy for the purpose of insurance upon the cargo (Act No. 2427, sec. 106).

Filipino Merchants Insurance v CA G.R. No. 85141 November 28, 1989


J. Regalado Facts: Choa insured 600 tons of fishmeal for the sum of P267,653.59 from Bangkok, Thailand to Manila against all risks under warehouse to warehouse terms. What was imported in the SS Bougainville was 59.940 metric tons at $395.42 a ton. The cargo was unloaded from the ship and 227 bags were found to be in bad condition by the arrastre. Choa made a formal claim against the defendant Filipino Merchants Insurance Company for P51,568.62 He also presented a claim against the ship, but the defendant Filipino Merchants Insurance Company refused to pay the claim. The plaintiff brought an action against the company and presented a third party complaint against the vessel and the arrastre contractor. The court below, after trial on the merits, rendered judgment in favor of private respondent, for the sum of P51,568.62 with interest at legal rate. The common carrier, Compagnie, was ordered to pay as a joint debtor. On appeal, the respondent court affirmed the decision of the lower court insofar as the award on the complaint is concerned and modified the same with regard to the adjudication of the thirdparty complaint. A motion for reconsideration of the aforesaid decision was denied. The AC made Filipino Merchants pay but absolved the common carrier, Compagnie. Hence this petition. Issues: 1. WON the "all risks" clause of the marine insurance policy held the petitioner liable to the private respondent for the partial loss of the cargo, notwithstanding the clear absence of proof of some fortuitous event, casualty, or accidental cause to which the loss is attributable. 2. WON The Court of Appeals erred in not holding that the private respondent had no insurable interest in the subject cargo, hence, the marine insurance policy taken out by private respondent is null and void.

Held: No. No. Petition denied. Ratio: 1. The "all risks clause" of the Institute Cargo Clauses read as follows: 5. This insurance is against all risks of loss or damage to the subject-matter insured but shall in no case be deemed to extend to cover loss, damage, or expense proximately caused by delay or inherent vice or nature of the subject-matter insured. Claims recoverable hereunder shall be payable irrespective of percentage. An "all risks policy" should be read literally as meaning all risks whatsoever and covering all losses by an accidental cause of any kind. Accident is construed by the courts in their ordinary and common acceptance. The very nature of the term "all risks" must be given a broad and comprehensive meaning as covering any loss other than a willful and fraudulent act of the insured. This is pursuant to the very purpose of an "all risks" insurance to give protection to the insured in those cases where difficulties of logical explanation or some mystery surround the loss or damage to property. Institute Cargo Clauses extends to all damages/losses suffered by the insured cargo except (a) loss or damage or expense proximately caused by delay, and (b) loss or damage or expense proximately caused by the inherent vice or nature of the subject matter insured. Generally, the burden of proof is upon the insured to show that a loss arose from a covered peril, but under an "all risks" policy the burden is not on the insured to prove the precise cause of loss or damage for which it seeks compensation. The insured under an "all risks insurance policy" has the initial burden of proving that the cargo was in good condition when the policy attached and that the cargo was damaged when unloaded from the vessel. The burden then shifts to the insurer to show the exception to the coverage. This creates a special type of insurance which extends coverage to risks not usually contemplated and avoids putting upon the insured the burden of establishing that the loss was due to the peril falling within the policy's coverage; the insurer can avoid coverage upon demonstrating that a specific provision expressly excludes the loss from coverage. Under an 'all risks' policy, it was sufficient to show that there was damage occasioned by some accidental cause of any kind, and there is no necessity to point to any particular cause. 2. Section 13 of the Insurance Code- anyone has an insurable interest in property who derives a benefit from its existence or would suffer loss from its destruction Insurable interest in property may consist in (a) an existing interest; (b) an inchoate interest founded on an existing interest; or (c) an expectancy, coupled with an existing interest in that out of which the expectancy arises. Choa, as vendee/consignee of the goods in transit, has such existing interest as may be the subject of a valid contract of insurance. His interest over the goods is based on the perfected contract of sale. The perfected contract of sale between him and the shipper of the goods operates to vest in him an equitable title even before delivery or before conditions have been performed. Further, Article 1523 of the Civil Code provides that where, in pursuance of a contract of sale, the seller is authorized or required to send the goods to the buyer, delivery of the goods to a carrier, for the purpose of transmission to the buyer is deemed to be a delivery of the goods to the buyer. The Court has heretofore ruled that the delivery of the goods on board the carrying vessels partake of the nature of actual delivery since, from that time, the foreign buyers assumed the risks of loss of the goods and paid the insurance premium covering them.

Roque vs. iac

FACTS: Manila Bay Lighterage Corporation (Manila Bay) a common carrier, entered into a contract with petitioners whereby the former would load and cary on board its barge Marble 10 about 422.18 cubic meters of logs from Malampaya Sound, Palawan to North Harbor Manila. The petitioners insured the logs against loss for 100,000.00 with respondent Pioneer Insurance and Surety Corporation (Pioneer). The petitioner loaded on the barge, 811 pieces of logs at Malampaya Sound, Palawan for carriage and delivery to North Harbor, Port of Manila, but the shipment never reached its destination because Marble 10 sank with the 811 pieces of logs somewhere off Cabuli Point in Palawan on its way to Manila. As alleged by the petitioners in their complaint and as found by both the trial and appellate courts, the barge where the logs were loaded was not seaworthy such that it developed a leak. The appellate court further found that one of the hatches was left open causing water to enter the barge and because the barge was not provided with the necessary cover or tarpauline, the ordinary splash of seawaves brought more water inside the barge. Respondent ignored the petitioner demand for payment of 150,000.00 for the loss of the shipment plus 100,000.00 as unrealized profits. Respondent Pioneer denied the claim of petitioner for the full amount of 100,000.00 on the ground that its liability depended upon the total loss of vessel only. The trial court decided in favor of the plaintiff (petitioner). The appellate court modified the trial courts decision and absolved Pioneer from liability after finding that there was a breach of implied warranty of seaworthiness on the part of the petitioners and that the loss of the insured cargo was caused by the perils of the ship and not by the perils of sea. It ruled that the loss is not covered by the marine insurance policy. ISSUE: Whether or not the implied warranty of seaworthiness in marine insurance attaches to the shipper who is not the shipowner. HELD: Section 113 of the Insurance Code provides: In every marine insurance upon a ship or freight, or freightage, or upon any thing which is the subject of marine insurance, a warranty is implied that the ship is seaworthy. Section 99 of the same Code also provides in part.

Marine insurance includes: (1) Insurance against loss of or damage to: (a) Vessels, craft, aircraft, vehicles, goods, freights, cargoes, merchandise, From the above-quoted provisions, there can be no mistaking the fact that the term cargo can be the subject of marine insurance and that once it is so made, the implied warranty of seaworthiness immediately attaches to whoever is insuring the cargo whether he be the shipowner or not. Moreover, the fact that the unseaworthiness of the ship was unknown to the insured is immaterial in ordinary marine insurance and may not be used by him as a defense in order to recover on the marine insurance policy. Since the law provides for an implied warranty of seaworthiness in every contract of ordinary marine insurance, it becomes the obligation of a cargo owner to look for a reliable common carrier which keeps its vessels in seaworthy condition. The shipper of cargo may have no control over the vessel but he has full control in the choice of the common carrier that will transport his goods. Or the cargo owner may enter into a contract of insurance which specifically provides that the insurer answers not only for the perils of the sea but also provides for coverage of perils of the ship. There is no doubt that the term perils of the sea extends only to losses caused by sea damage, or by the violence of the elements, and does not embrace all losses happening at sea. They insure against losses from extraordinary occurrences only, such as stress of weather, winds and waves, lightning, tempests, rocks and the like. These are understood to be the perils of the sea referred in the policy, and not those ordinary perils which every vessel must encounter. Perils of the sea has been said to include only such losses as are of extraordinary nature, or arise from some overwhelming power, which cannot be guarded against by the ordinary exertion of human skill and prudence. Damage done to a vessel by perils of the sea includes every species of damages done to a vessel at sea, as distinguished from the ordinary wear and tear of the voyage, and distinct from injuries suffered by the vessel in consequence of her not being seaworthy at the outset of her voyage (as in this case). It is also the general rule that everything which happens thru the inherent vice of the thing, or by the act of the owners, master or shipper, shall not be reputed a peril, if not otherwise borne in the policy. On the contention of the petitioners that the trial court found that the loss was occasioned by the perils of the sea characterized by the storm and waves which buffeted the vessel, the records show that the court ruled otherwise. It stated: x x x The other affirmative defense of defendant Lighterage, That the supposed loss of the logs was occasioned by force majeure was not supported by the evidence. At the time Mable 10 sank, there was no typhoon but ordinary strong wind and waves, a condition which is natural and normal in the open sea. The evidence shows that the sinking of Mable 10 was due to improper loading of the logs on one side so that the barge was tilting on one side and for that it did not navigate on even keel; that it was no longer seaworthy that was why it developed leak; that the personnel of the tugboat and the barge

committed a mistake when it turned loose the barge from the tugboat east of Cabuli point where it was buffeted by storm and waves, while the tugboat proceeded to west of Cabuli point where it was protected by the mountain side from the storm and waves coming from the east direction. x x x lt must be considered to be settled, furthermore, that a loss which, in the ordinary course of events, results from the natural and inevitable action of the sea, from the ordinary wear and tear of the ship, or from the negligent failure of the ships owner to provide the vessel with proper equipment to convey the cargo under ordinary conditions, is not a peril of the sea. Such a loss is rather due to what has been aptly called the peril of the ship. The insurer undertakes to insure against perils of the sea and similar perils, not against perils of the ship. As was well said by Lord Herschell in Wilson, Sons & Co. v. Owners of Cargo per the Xantho ([1887], 12 A. C., 503, 509), there must, in order to make the insurer liable, be some casualty, something which could not be foreseen as one of the necessary incidents of the adventure. The purpose of the policy is to secure an indemnity against accidents which may happen, not against events which must happen. Barratry as defined in American Insurance Law is any willful misconduct on the part of master or crew in pursuance of some unlawful or fraudulent purpose without the consent of the owners, and to the prejudice of the owners interest, (Sec. 171, U.S. Insurance Law, quoted in Vance, Handbook on Law of Insurance, 1951, p. 929.) Barratry necessarily requires a willful and intentional act in its commission. No honest error of judgment or mere negligence, unless criminally gross, can be barratry. (See Vance on Law of Insurance, p. 929 and cases cited therein.) In the case at bar, there is no finding that the loss was occasioned by the willful or fraudulent acts of the vessels crew. There was only simple negligence or lack of skill. Hence, the second assignment of error must likewise be dismissed.
THE PHILIPPINE AMERICAN GENERAL INSURANCE COMPANY INC v. CA (FELMAN SHIPPING LINES) 273 SCRA 226 BELLOSILLO; June 11, 1997 FACTS - Coca-Cola Bottlers Philippines, Inc., loaded on board MV Asilda, a vessel owned and operated by Felman 7,500 cases of 1-liter Coca-Cola softdrink bottles to be transported from Zamboanga City to Cebu for consignee Coca-Cola Bottlers Philippines, Inc., Cebu. The shipment was insured with petitioner Philippine American General under Marine Open Policy. - The vessel sank in the waters of Zamboanga del Norte bringing down her entire cargo with her including the subject 7,500 cases of 1-liter Coca-Cola softdrink bottles. - The consignee filed a claim with respondent FELMAN for recovery of damages it sustained as a result of the loss of its softdrink bottles that sank with MV Asilda. Respondent denied the claim thus prompting the consignee to file an insurance claim with PHILAMGEN which paid its claim of P755,250.00. - Claiming its right of subrogation PHILAMGEN sought recourse against respondent FELMAN which disclaimed any liability for the loss. Consequently, PHILAMGEN sued the shipowner for sum of money and damages. - PHILAMGEN alleged that the sinking and total loss of MV Asilda and its cargo were due to the vessels unseaworthiness as she was put to sea in an unstable condition. It further alleged that the vessel was improperly manned and that its officers were grossly negligent in failing to take appropriate measures to proceed to a nearby port or beach after the vessel started to list. - FELMAN filed a motion to dismiss based on the affirmative defense that no right of subrogation in favor of PHILAMGEN was transmitted by the shipper, and that, in any event, FELMAN had abandoned all its rights, interests and ownership over MV Asilda together with her freight and appurtenances for the purpose of limiting and extinguishing its liability under Art. 587 of the Code of Commerce. - Trial court dismissed the complaint of PHILAMGEN. On appeal the Court of Appeals set aside the dismissal and

remanded the case to the lower court for trial on the merits. FELMAN filed a petition for certiorari with this Court but it was subsequently denied on 13 February 1989. - Trial court rendered judgment in favor of FELMAN. It ruled that MV Asilda was seaworthy when it left the port of Zamboanga as confirmed by certificates issued by the Philippine Coast Guard and the shipowners surveyor attesting to its seaworthiness. Thus the loss of the vessel and its entire shipment could only be attributed to either a fortuitous event, in which case, no liability should attach unless there was a stipulation to the contrary, or to the negligence of the captain and his crew, in which case, Art. 587 of the Code of Commerce should apply. - CA ruled that MV Asilda was unseaworthy for being top- heavy as 2,500 cases of Coca-Cola softdrink bottles were improperly stowed on deck. Nonetheless, the appellate court denied the claim of PHILAMGEN on the ground that the assureds implied warranty of seaworthiness was not complied with. Perfunctorily, PHILAMGEN was not properly subrogated to the rights and interests of the shipper. Furthermore, respondent court held that the filing of notice of abandonment had absolved the shipowner/agent from liability under the limited liability rule. ISSUES 1. WON MV Asilda was seaworthy when it left the port of Zamboanga 2. WON the limited liability under Art. 587 of the Code of Commerce should apply 3. WON PHILAMGEN was properly subrogated to the rights and legal actions which the shipper had against FELMAN, the shipowner HELD 1. YES - MV Asilda was unseaworthy when it left the port of Zamboanga. We subscribe to the findings of the Elite Adjusters, Inc., and the Court of Appeals that the proximate cause of the sinking of MV Asilda was its being topheavy. Contrary to the ship captains allegations, evidence shows that approximately 2,500 cases of softdrink bottles were stowed on deck. Several days after MV Asilda sank, an estimated 2,500 empty Coca-Cola plastic cases were recovered near the vicinity of the sinking. Considering that the ships hatches were properly secured, the empty Coca-Cola cases recovered could have come only from the vessels deck cargo. It is settled that carrying a deck cargo raises the presumption of unseaworthiness unless it can be shown that the deck cargo will not interfere with the proper management of the ship. However, in this case it was established that MV Asilda was not designed to carry substantial amount of cargo on deck. The inordinate loading of cargo deck resulted in the decrease of the vessels metacentric height thus making it unstable. The strong winds and waves encountered by the vessel are but the ordinary vicissitudes of a sea voyage and as such merely contributed to its already unstable and unseaworthy condition. 2. NO - The ship agent is liable for the negligent acts of the captain in the care of goods loaded on the vessel. This liability however can be limited through abandonment of the vessel, its equipment and freightage as provided in Art. 587. Nonetheless, there are exceptional circumstances wherein the ship agent could still be held answerable despite the abandonment, as where the loss or injury was due to the fault of the shipowner and the captain. The international rule is to the effect that the right of abandonment of vessels, as a legal limitation of a shipowners liability, does not apply to cases where the injury or average was occasioned by the shipowners own fault. 3. YES - The doctrine of subrogation has its roots in equity. It is designed to promote and to accomplish justice and is the mode which equity adopts to compel the ultimate payment of a debt by one who in justice, equity and good conscience ought to pay. Therefore, the payment made by PHILAMGEN to Coca-Cola Bottlers Philippines, Inc., gave the former the right to bring an action as subrogee against FELMAN. Having failed to rebut the presumption of fault, the liability of FELMAN for the loss of the 7,500 cases of 1-liter Coca-Cola softdrink bottles is inevitable. - Sec. 113 of the Insurance Code provides that (i)n every marine insurance upon a ship or freight, or freightage, or upon anything which is the subject of marine insurance, a warranty is implied that the ship is seaworthy. Under Sec. 114, a ship is seaworthy when reasonably fit to perform the service, and to encounter the ordinary perils of the voyage, contemplated by the parties to the policy. Thus it becomes the obligation of the cargo owner to look for a reliable common carrier which keeps its vessels in seaworthy condition. He may have no control over the vessel but he has full control in the selection of the common carrier that will transport his goods. He also has full discretion in the choice of assurer that will underwrite a particular venture. - In policies where the law will generally imply a warranty of seaworthiness, it can only be excluded by terms in writing in the policy in the clearest language. And where the policy stipulates that the seaworthiness of the vessel as between the assured and the assurer is admitted, the question of seaworthiness cannot be raised by the assurer without showing concealment or misrepresentation by the assured. - PHILAMGENs action against FELMAN is squarely sanctioned by Art. 2207 of the Civil Code which provides: Art. 2207. If the plaintiffs property has been insured, and he has received indemnity from the insurance company for the injury or loss arising out of the wrong or breach of contract complained of, the insurance company shall be subrogated to the rights of the insured against the wrongdoer or the person who has violated the contract. If the amount paid by the insurance company does not fully cover the injury or loss, the aggrieved party shall be entitled to recover the deficiency from the person causing the loss or injury. Disposition Petition is GRANTED. Respondent FELMAN SHIPPING LINES is ordered to pay petitioner PHILIPPINE AMERICAN GENERAL INSURANCE CO., INC., Seven Hundred Fifty-five Thousand Two Hundred and Fifty Pesos (P755,250.00) plus legal interest thereon counted from 29 November 1983, the date of judicial demand, pursuant

to Arts. 2212 and 2213 of the Civil Code.

You might also like